Sei sulla pagina 1di 58

# DAILY QUIZ

#www.insightsias.com
#www.insightsonindia.com
|

*Clarifications* also generate hydropower” which is wrong.


Statement 2 which says, “associated with
Narmada” is correct. Statement 3 says,
“filling….dams in Maharashtra’s Vidarbha
Quiz, August 30th (Click Here): region” which is wrong. Therefore, the
incorrect statements are 1 and 3. Answer
Q5. Regarding nations which have remains the same, viz. option ‘c’.
successfully tested the scramjet
technology: This is why, the word
“nation” was mentioned in italics here, as well
as in the question. The European Space Agency Q5. Question related to “Prakampana”: Look
does not belong to one nation. It is an at the options; Option ‘c’ (which was supposed
intergovernmental organisation. Space to contain the correct statement) is missing!
agencies of India, Russia, China, USA and ESA This was a mistake from our side while
have successfully tested SCRAMJET uploading the questions. If you read the
technology, and India is the fourth explanation, the answer is clear. The first
nation among them. Answer remains the sentence in the explanation says, “The maiden
same, so does the explanation given under the Joint Disaster Management Exercise named
solution. Prakampana (‘Cyclone’ in Sanskrit) aimed at
synchronising resources and efforts of all
agencies involved in disaster management has
Quiz, August 31st (Click Here): been inaugurated at Visakhapatnam today,
30th August 16.”

Q1. Question Regarding LEMOA – The specific


question for Statement 1 would be, “which all
nations under this agreement will get access Q6. Totaliser Machines related
to Indian military facilities for supplies and question: Answer remains the same. Option
repairs?” ‘a’ is correct, so is option ‘c’. Therefore, option
The agreement says, “The LEMOA will give the ‘d’ which says “both a and c” is the correct
militaries of both countries access to each option. Please read the explanation.
other’s facilities for supplies and repairs.” This
means two things – Under this particular
agreement, A humble request – First, please read the
 India will be the only country to be able to questions carefully! Many tend to skip the
access US facilities for supplies and repairs. directive part of the question (correct, NOT
 US will be the only country to be able to correct, incorrect etc). Second, please read the
access Indian facilities for supplies and explanations carefully. If you still have any
repairs. queries and can spare an additional 2-3
The answer will remain the same, that is minutes, go through the associated articles –
option ‘b’ – 1 and 2 only. for this, click on the hyperlinks(s) and/or the
website link(s) provided. If you still have any
queries, comment down below (then, hit the
like button, share and subscribe! :-D)
Q4. Sauni Yojana question: Please read the
question carefully. The directive of the
question is to select the “incorrect”
statements. Statement 1 says, “….which will

©www.InisightsIAS.com 1 ©www.InsightsonIndia.com
|

1. The Union Cabinet recently gave its approval (a) Sectarian


to create a Project Development Fund (PDF)
with a corpus of Rupees 500 crores for (b) Communal
catalysing Indian economic presence in which (c) Political
of the following nations?
(d) Agrarian
1. Myanmar

2. Cambodia
Solution: d.
3. Laos
http://www.insightsonindia.com/2016/08/3
4. Vietnam 1/insights-daily-current-affairs-31-august-
Select the correct answer using the code given 2016/
below: http://www.thehindu.com/news/national/3
(a) 1 only 27-rise-in-agrarian-riots-in-2015-ncrb-report-
shows/article9051348.ece
(b) 2, 3 and 4 only

(c) 1 and 4 only


3. Two Indians are among the winners of the
(d) 1, 2, 3 and 4 Ramon Magsaysay Awards 2016. For what
services of theirs have they been bestowed
with this prestigious award?
Solution: d. 1. Anti-corruption efforts
“CLMV countries namely Cambodia, Laos, 2. Efforts to eradicate manual scavenging
Myanmar and Vietnam have a unique
position in the regional value chains and offer 3. Using Carnatic music to break the barriers
a gateway for market access to China/EU and of caste and class
other markets due to various trade
agreements. The key advantage of positioning 4. Championing the rights of tribal people
India on the regional value chains is securing through written works and legal battles
on a long term basis, a dedicated market for Select the correct answer using the code given
Indian raw materials and intermediate goods below:
besides a dedicated source for inputs and raw
materials for Indian industry. While (a) 1 and 3
opportunities are a plenty in CLMV region,
(b) 1 and 4
Indian entrepreneurs’ endeavors in these
countries have, thus far, been limited due to (c) 2 and 3
limited information, infrastructure and other
contingent risks. The PDF shall benefit India’s (d) 2 and 4
industrial community for business expansion,
and to maintain cost competitive supply
chains, besides integrating with global Solution: c.
production networks.”
Sanjiv Chaturvedi is an Indian Forest Service
http://pib.nic.in/newsite/PrintRelease.aspx? officer and a whistleblower who received the
relid=149378 award in 2015. Statement 4 refers to
Mahasweta Devi who recently passed away
and received the award in 1997.
2. The recently released report “Crime in India”
http://www.thehindu.com/news/national/be
for 2015 by the National Crime Records
zwada-wilson-tm-krishna-receive-magsaysay-
Bureau noted that the recorded incidents of
award/article9056484.ece
which form of riots jumped 327 per cent from
2014 to 2015?

©www.InisightsIAS.com 2 ©www.InsightsonIndia.com
|

4. With reference to the Ganges River Dolphin, http://www.thehindu.com/news/national/ot


consider the following statements: her-states/it-is-pollution-that-made-gangetic-
dolphins-blind-says-uma-
1. It uses its eye to locate itself. bharti/article9056507.ece
2. Because of the sound it produces when
breathing, the animal is popularly referred
to as the ‘Susu’. 5. With reference to the National Council of
Senior Citizens, which of the following
3. It is at the apex of the aquatic food chain, statements is/are correct?
and its presence in adequate numbers
symbolises greater biodiversity in the river 1. It is chaired by the Minister of Social
system. Justice and Empowerment.

Which of the statements given above is/are 2. The council is mandated to meet at least
incorrect? twice a year.

(a) 1 only 3. It was originally known as the National


Council for Older Persons which was set
(b) 1 and 2 only up in 2012.
(c) 2 and 3 only Select the correct answer using the code given
(d) None of the above below:

(a) 1 and 2 only

Solution: d. (b) 2 and 3 only

“Evolutionary biology has it that the Gangetic (c) 1 and 3 only


dolphin has been ‘blind’ for about 20 million (d) 1, 2 and 3
years and shares its limited eyesight with its
international freshwater river-dolphin
cousins, namely the Yangtze, Amazon, La
Plata and the Indus dolphins.” Solution: a.

“The Gangetic dolphin lost its eyes in the “The National Policy on Older Persons
course of evolution to adapt to the muddy (NPOP), 1999 envisages State support to
water of rivers. They largely navigate by echo- ensure financial and food security, health
location or sonar. The mammal evolved over care, shelter and other needs of older persons,
millennia, according to an October 2000 paper equitable share in development, protection
in the Proceedings of the Royal Society of against abuse and exploitation, and
London, Evolution of river dolphins, when availability of services to improve the quality of
rising sea levels inundated large areas of the their lives. The policy also covers issues like
Indo-Gangetic basin, creating a shallow social security, intergenerational bonding,
marine habitat.” family as the primary caretaker, role of Non-
Governmental Organizations, training of
However, according to the WWF: Ganges River manpower, research and training.”
Dolphin – The dolphin has the peculiarity of
swimming on one side so that its flipper trails “The National Council of Older Persons was
the muddy bottom. This behaviour is constituted in 1999 to monitor the
understood to help it to find food. Being a implementation of the Policy and advise the
mammal, the Ganges River dolphin cannot Government on issues related to the welfare of
breathe in the water and must surface every senior citizens.”
30-120 seconds. Although its eye lacks a lens http://www.insightsonindia.com/2016/08/3
(this species is also referred to as the “blind 1/insights-daily-current-affairs-31-august-
dolphin”), the dolphin still uses its eye to locate 2016/
itself.
About National Policy and Council on/of
Older Persons/Senior Citizens:

©www.InisightsIAS.com 3 ©www.InsightsonIndia.com
|

http://pib.nic.in/newsite/PrintRelease.aspx? “The Ministry of Home Affairs had constituted


relid=108092 a committee under the chairmanship of former
home secretary Madhukar Gupta in April
2016, for strengthening border protection and
6. The recently released report “Crime in India” addressing the issue of gaps and vulnerability
for 2015 by the National Crime Records in border fencing along the India-Pakistan
Bureau noted that the recorded incidents of border. The mandate of the committee was to
which form of riots jumped 327 per cent from study all types of gaps in fencing and all other
2014 to 2015? vulnerabilities in the International Border and
to suggest comprehensive approach to fix
(a) Sectarian these gaps in fencing and other vulnerabilities
on interim and permanent basis.”
(b) Communal
http://www.insightsonindia.com/2016/08/3
(c) Political
1/insights-daily-current-affairs-31-august-
(d) Agrarian 2016/

Solution: d.

http://www.insightsonindia.com/2016/08/3 Notes
1/insights-daily-current-affairs-31-august-
2016/

http://www.thehindu.com/news/national/3
27-rise-in-agrarian-riots-in-2015-ncrb-report-
shows/article9051348.ece

7. The mandate of the Madhukar Gupta


committee, which recently submitted its
report, was to

(a) Only study all types of gaps in fencing


along the international border and to
suggest comprehensive approach to fix
these gaps in fencing.

(b) Study all types of gaps in fencing and all


other vulnerabilities along the
International Border.

(c) Study the feasibility of laser fencing for all


gaps in fencing along the International
Border.

(d) Study all types of gaps in fencing and all


other vulnerabilities along the
international border between India and
Pakistan on the one hand and Bangladesh
on the other.

Solution: b.

Statement ‘b’ is incomplete but the most


appropriate one among the four given above.

©www.InisightsIAS.com 4 ©www.InsightsonIndia.com
|

1. The phrase ‘eminent domain’ is sometimes Statement ‘a’ refers to the concept of
seen in the news with respect to government ‘Procedure established by Law’. Statement ‘c’
actions/policies. This means that the refers to ‘Rule of Law’.
Government
“The lesson from Singur is that thoughtless
(a) Can expropriate private property for sale to and fast-tracked acquisition, often to the
private entities if the end use is for the detriment of due process and the interests of
benefit of the public. those deprived of land and livelihood, is the
real issue — and not promotion of
(b) is not answerable to the judiciary for its industrialisation.”
actions in certain land acquisition-related
matters; It is a concept embedded in http://www.thehindu.com/opinion/editorial/
Schedule 9 of the Indian Constitution. full-circle-in-singur/article9061088.ece

(c) has a right to expropriate private property


for public use.
3. Consider the following pairs:
(d) None of the above.
Critically Endemic to
Endangered Location(s)
Species
Solution: c.
1. Red Sanders : Andhra
Most appropriate between option ‘a’ and ‘c’ is Pradesh and
the latter. Tamil Nadu

“The Supreme Court’s verdict on the Singur


2. Beddome’s Cycas : Maharashtra
land acquisition issue is a re-statement of first
principles relating to the limitations of
‘eminent domain’. By quashing the entire land 3. Himalayan Yew : North East
acquisition process done by the erstwhile Left India
Front government in West Bengal, the court Which of the pairs mentioned above is/are
has reiterated that the term ‘public purpose’ correctly matched?
cannot be arbitrarily invoked to acquire land
(a) 1 only
and hand it over to a private party.”
(b) 2 and 3 only
http://www.thehindu.com/opinion/editorial/
full-circle-in-singur/article9061088.ece (c) 1 and 3 only

(d) 1, 2 and 3
2. Which of the following statements most
appropriately defines the legal concept of ‘Due
Process’? Solution: c.

(a) The state must follow the procedure that Beddome’s Cycas is endemic to Andhra
has been established as per legislations. Pradesh. (According to IUCN though, it is
endangered, not Critically Endangered. Ditto
(b) The state must respect all legal rights that is the case with the Himalayan Yew as well as
are owed to a person. Red Sanders. Prefer the IUCN categorisation
over the PIB article categorisation.
(c) Laws should govern a state, and not the
Additionally, the Himalayan Yew is also found
arbitrary decisions of the executive.
in Nepal, South West China, Philippines, etc.
(d) None of the above and is not endemic to India as stated in the
article.)

PIB Features: Scroll down to view images of the


Solution: b. list of critically endangered species (as stated
in the article): “Conserving the Medicinal
Plants”, August 31st, 2016

©www.InisightsIAS.com 5 ©www.InsightsonIndia.com
|

Over the next few days we will cover articles 5. The National Board for Wildlife is chaired by
that appeared in August 2016 in PIB features.
We will try providing extracts from them, (a) The Prime Minister
which are most relevant for our purpose, here (b) The Minister of Environment, Forest and
in the form of questions and explanations. Climate Change (MoEFCC)

(c) An eminent individual appointed by a


4. Consider the following statements with committee consisting of the PM, the
reference to the TRAFFIC programme, of which MoFECC and leader of opposition from the
the World Wide Fund for Nature and the Lok Sabha
International Union for Conservation of Nature (d) An eminent individual appointed by a
and Natural Resources are its partner committee consisting of members of the
organisations: Union Cabinet and the MoEFCC
1. Its mission is to ensure that trade in wild
animals and plants is not a threat to the
conservation of nature. Solution: a.

2. It is a non-governmental organisation. The National Board for Wildlife is chaired by


India’s Prime Minister and its vice chairman
Which of the statements given above is/are is Minister of Environment. Further, the
correct? board is mammoth body with 47-members
(a) 1 only including Parliament Members, NGOs,
eminent conservationists, ecologists and
(b) 2 Only environmentalists, Government secretaries of
various departments, Chief of the Army Staff,
(c) Both 1 and 2 Director General of Forests, tourism etc. etc.
(d) Neither 1 nor 2
 The members include 15 non-government
members, 19 ex-officio members and 10
government officials such as secretaries.
Solution: c.  There is a standing committee of the board
which approves all the projects falling
“The Indian chapter of World Wildlife Fund
within protected wildlife areas or within 10
TRAFFIC has deliberated over standards and
km of them; but notably, law does not
certification schemes in medicinal and
specify the formation of the standing
aromatic plants in India. India has about
committee. The rules under the wildlife law
45,000 plant species and over a sixth of them
allow the board to take decisions with half
standing at 7,333 are medicinal aromatic
the members but are silent about the
plants. But only 15% of medicinal plants are
standing committee.
cultivated while the remaining 85% are
collected by the industry from forest http://www.thehindu.com/opinion/columns
ecosystems and other natural habitats. Dr S K /our-compromised-ecological-
Niraj, spearheading TRAFFIC is apprehensive security/article9061267.ece
of the glaring threat of mass extinction of 3000
medicinal plants of India in the face of
escalating commercial demand.” Notes
Improvisation: PIB Features – “Conserving the
Medicinal Plants”, August 31st, 2016

http://www.traffic.org/overview/

©www.InisightsIAS.com 6 ©www.InsightsonIndia.com
|

1. Article 21A of the Indian Constitution is 3. The Ministry of Shipping, apart from
associated with developing port facilities within the nation, is
in the process of developing/has developed
(a) Fundamental Duties ports of which of the following nations?
(b) The Right to Education 1. Chabahar, Iraq
(c) Protection of Life and Personal Liberty 2. Salalah, Oman
(d) Right to Constitutional Remedies 3. Sittwe Port, Myanmar

Select the correct answer using the code given


Solution: b. below:

“Every child between the ages of 6 to 14 years (a) 1 and 3 only


has the right to free and compulsory (b) 2 and 3 only
education. This is stated as per the 86th
Constitution Amendment Act via Article (c) 3 only
21A. The Right to Education Act seeks to give
effect to this amendment.” (d) None of the above

http://indiacode.nic.in/coiweb/amend/amen
d86.htm Solution: d.
http://pib.nic.in/newsite/PrintRelease.aspx? “For the first time, the Ministry is developing
relid=149495 port facilities outside the country at Chabahar
in Iran (not Iraq).”

2. “Providing education to children” is provided The Sittwe Port is being developed, as a part
for in which of the following sections of the of the Kaladan Multi-Modal Transit
Indian Constitution? Transport Project, by India through a private
entity and not the Ministry of Shipping.
1. Fundamental Rights
http://pib.nic.in/newsite/PrintRelease.aspx?
2. Directive Principles of State Policy relid=149475

3. Fundamental Duties

Select the correct answer using the code given 4. There has been some talk of the “World Land
below: Bridge” as a “natural sequel” to the Chinese
Belt and Road connectivity Initiative. This
(a) 1 and 2 only “Bridge”, if realised, will directly link
(b) 2 and 3 only (a) Hawaii and China
(c) 1 and 3 only (b) Alaska and China
(d) 1, 2 and 3 (c) North America and Russia

(d) North America and Europe via Greenland


Solution: d.

Article 21A, 45 and 51A contain references to Solution: c.


providing education to children.
“China’s Belt and Road connectivity initiative,
Improvisation: which bears a strong imprint of the Eurasian
http://pib.nic.in/newsite/PrintRelease.aspx? Land Corridor – a blueprint conceived by The
relid=149495 Schiller Institute – should be followed by a
http://indiacode.nic.in/coiweb/amend/amen World Land Bridge that will link North
d86.htm America with the New Silk Road, says the

©www.InisightsIAS.com 7 ©www.InsightsonIndia.com
|

co-founder of a top think-tank. In an interview


with The Hindu, ahead of the G-20 summit in *Clarifications*
Hangzhou, Helga Zepp-LaRouche, the co-
founder of The Schiller Institute (TSI) says the Q4, September 1st Quiz (Click Here): The Ganges
World Land Bridge is the natural sequel to dolphin is not completely blind. Refer to paragraph 1
the Eurasian Land Bridge, the mega-
(which mentions, “limited eyesight”) and 3 (which
connectivity initiative to revive the ancient Silk
mentions that “it still uses its eye to locate itself”) in
Road in all its dimensions, including its lost
the explanation. Paragraph 2 explains how the dolphin
cultural and civilizational attributes.”
manages with poor eyesight (although its mention of
http://www.thehindu.com/news/internation “lost its eyes in the course of evolution to adapt to the
al/interview-with-helga-zepplarouche- muddy water of rivers” is slightly misleading because it
cofounder-of-the-schiller- hasn’t lost its eyes! Its eyesight is very poor, and it still
institute/article9066424.ece has its eyes, see any image of the Ganges dolphin.)

5. Consider the following statements: Q7, September 1st Quiz (Click Here): None of the
1. The right to reputation is a constituent of options are correct. The correct answer was supposed
Article 21 of the Constitution. to be option ‘b’, but we made a mistake while framing
the statement. The statement should have been “Study
2. Section 499 of the Indian Penal Code all types of gaps in fencing and all other vulnerabilities
prescribes the punishment for a person along the India-Pakistan border” rather than, “along
and his family found guilty of demanding
the international border” as was mentioned. Option ‘a’
dowry.
is incorrect because the committee was mandated not
Which of the statements above is/are correct? “only” for studying all types of gaps and suggesting
comprehensive approaches to fix gaps in fencing, but
(a) 1 Only
also to address vulnerabilities other than gaps in
(b) 2 Only border fencing along India-Pakistan border.

(c) Both 1 and 2

(d) Neither 1 nor 2 NOTES

Solution: a.

“The court held that criminalisation of


defamation to protect individual dignity of
life and reputation is a “reasonable
restriction” on the fundamental right of
free speech and expression. “The right to
reputation is a constituent of Article 21 of the
Constitution. It is an individual’s fundamental
right,” Justice Misra observed.”

Section 498A of the IPC is the anti-dowry


harassment law, Sections 499 & 500 are
associated with criminal defamation.

Improvisation:
http://indianexpress.com/article/opinion/co
lumns/defamation-is-no-crime-3010691/

http://www.thehindu.com/todays-paper/sc-
upholds-law-on-criminal-
defamation/article8597865.ece

©www.InisightsIAS.com 8 ©www.InsightsonIndia.com
|

1. “Bird Fancier’s Lung” is an inflammatory lung Select the correct answer using the code given
condition caused by below:

(a) PM2.5-sized dust particles gathered and (a) 1 only


released into the air due to regular wing-
flapping by birds. (b) 1 and 2 only

(b) Bird droppings. (c) 2 and 3 only

(c) A bacteria known to biologists as “LFB” (d) 1 and 3 only


that resides in feathers of pigeons but
infects only human lungs.
Solution: b.
(d) It is not an inflammatory lung condition; it
refers to a peculiar case in which smoker’s “In December 2015, a global framework to
lungs are not affected by the reach zero human rabies deaths by 2030 was
hypersensitivity that is caused among non- launched by WHO and the World
smokers due to PM2.5-sized dust particles. Organisation for Animal Health (OIE), in
collaboration with the Food and Agriculture
Organization of United Nations (FAO) and the
Solution: b. Global Alliance for Rabies Control. This
initiative marks the first time that the human
“When G.S. Srinivas took his mother, K. and animal health sectors have come together
Sitakumari, to a pulmonologist for a recurring to adopt a common strategy against this
breathing problem, the doctor’s line of devastating but massively neglected disease.”
questioning seemed bizarre. “Do you live in
close proximity to pigeons,” the doctor had The BMGF may be supporting it financially or
asked. When the answer was in the may be a participant in implementing the
affirmative, the doctor’s prescription was even rabies control programme, but the programme
stranger — put up a fish net and keep the hasn’t been launched in collaboration with
birds at bay. The remedy worked and Ms. BMGF.
Sitakumari recovered completely. The http://www.who.int/mediacentre/factsheets
pulmonologist, Dr. Vijay Kumar /fs099/en/
Chennamchetty, of Apollo Hospitals in
Hyderabad, had diagnosed what most Indian
pulmonologists will miss — Hypersensitive
3. Consider the following statements:
Pneumonitis (HP) or Bird Fancier’s Lung —
an inflammatory lung condition caused by bird 1. Sikkim is the only state in the country to
droppings that are highly allergenic.” have achieved the numbers mandated by
the WHO for sterilisation of stray dogs.
http://www.thehindu.com/sci-
tech/health/the-pigeon-paradox-feeding- 2. The WHO considers sustainable
them-could-be-bad-for-your- vaccination of ninety per cent of the at-risk
lungs/article9070475.ece dog population as a critical success factor
in the elimination of dog-mediated human
rabies.
2. In December 2015, a global framework to
Which of the statements above is/are correct?
reach zero human rabies deaths by 2030 was
launched by the World Health Organisation (a) 1 only
and the World Organisation for Animal Health
(OIE), in collaboration with (b) 2 only

1. Global Alliance for Rabies Control (c) Both 1 and 2

2. Food and Agriculture Organisation (d) Neither 1 nor 2

3. Bill and Melinda Gates Foundation

©www.InisightsIAS.com 9 ©www.InsightsonIndia.com
|

Solution: a. yellow-silver-red Jenu Kurumba men. They


are dressed as women, in a playful take on
The WHO considers sustainable vaccination of Western sexuality. They are walking to a forest
seventy per cent of the at-risk dog population temple to invert normality — they will not
as one among many critical success factors in worship but berate the gods.”
the elimination of dog-mediated human rabies.
“Liberated by disguise and alcohol, the Kunde
Annual mass anti-rabies vaccination program festival topples the norms — gods, passers-by
in Sikkim under the SARAH Programme and plantation owners are all cheerfully
(Sikkim Anti-Rabies and Animal Health abused by the labour force, as today they
Programme): “The mass dog vaccination stand upright rather than bent over the coffee
campaign teams aim to visit every household bushes. Kunde Habba in the local dialect
throughout Sikkim over a four-week period. means the festival of abusing god. Screams
Even remote villages with no road access are and laughter are in the air but this crazy
visited by foot to vaccinate dogs and deworm carnival serves a purpose — it’s a social
puppies. We have achieved at least 70% pressure cooker that allows everyone to let off
vaccination coverage, and in East Sikkim have steam. The static hierarchy of the hills is
achieved 80% coverage. Vaccinated dogs in the blown open for a day, which makes the rest of
border regions provide a buffer zone against the year more bearable for those near the
rabies re-introduction from neighbouring bottom.”
rabies-endemic areas.”
http://www.thehindu.com/features/magazin
“In Chennai, at least until two years ago, the e/clare-arni-captures-the-kurumba-men-of-
stray dog population had been brought under coorg-during-their-annual-festival-of-
control. The Jaipur and Ooty corporations catharsis/article9069708.ece
have controlled rabies to a large extent.
Seventy per cent of the stray dogs in Sikkim
have been sterilised, which is the only state
in the country to achieve the numbers 5. India has in recent times emerged as a major
mandated by the WHO.” player in providing disaster assistance to
countries in the Indian Ocean region, and, in
http://indianexpress.com/article/india/india line with that, storage of HADR ‘bricks’ on
-news-india/dog-bite-deaths-kerala-delhi- ships has now become a Standard Operating
india-dog-population-control-3012148/ Procedure and all operational ships carry it at
all times. What does “HADR” stand for?

(a) Hazard and Disaster Assistance and


4. The “Kunde festival” is a traditional festival of Rehabilitation
the tribal people in Coorg. The speciality of this
festival is (b) Hazard Aid and Disaster Relief

(a) The use of swear words against the gods. (c) High Availability Disaster Recovery

(b) A long day of fast by devotees followed by (d) Humanitarian and Disaster Relief
piercing of their bodies.

(c) The fire walking ceremony.


Solution: d.
(d) The trading of donkeys which are
decorated in various colours. “Navy frigate INS Trikand, which docked at a
Madagascar port as part of a goodwill visit to
East Africa and Southern Indian Ocean, has
handed over relief material for those affected
Solution: a. by the recent bush fires in that country. “In
“Once a year, the usual peace of the Coorg support of the government and the people of
hills is rent by expletives and abuse, the Madagascar, the Indian Navy swung into
staccato beat of plastic drums, and rhythmic immediate action to provide succour to more
chants and yells. Then, from the muted shades than five thousand affected citizens,” the Navy
of green and brown foliage emerge the neon- said in a statement. INS Trikand donated

©www.InisightsIAS.com 10 ©www.InsightsonIndia.com
|

essential food items and medicines to the government procurement, regulations and
affected, since the ship is always ready to competition was pushed.”
provide Humanitarian Assistance and
Disaster Relief [HADR] during such http://www.thehindu.com/news/national/rs
eventualities.” s-wings-go-hammer-and-tongs-at-bjps-fdi-
policy/article9070442.ece
“India has in recent times emerged as a major
player in providing disaster assistance to
countries in the region, and, in line with that,
storage of HADR ‘bricks’ on ships has now
Notes
become a Standard Operating Procedure (SOP)
and all operational ships carry it at all times.”

http://www.thehindu.com/news/national/in
s-trikand-hands-over-relief-material-to-
madagascar/article9070417.ece

6. “Singapore issues”, a phrase seen in the news


sometimes, refers to

(a) A United Nations report on Human and


Animal rights in Singapore which was
published following the massive dog-
culling programme.

(b) A set of issues raised and solutions


proposed by the Singapore Government
regarding over-extraction of maritime
resources by the South-East Asian nations
in their regional waters.

(c) Issues first raised by India at the WTO’s


Singapore Ministerial with regard to the
Special Safeguard Mechanism and Public
Stockholding norms.

(d) Four issues introduced to the WTO agenda


at the Ministerial Conference in Singapore,
one of which was trade facilitation.

Solution: d.

“Another co-convener, Dhanpat Ram Agarwal,


an expert on Intellectual Property Rights, also
flagged the “U-turn”. “The stand of the then
NDA government [in 2001] was that the
‘Singapore Issues’ on investment…[on]
transparency in government procurements
and competition [Trade Facilitation having
been accepted in the 9th Ministerial
Conference at Bali] will be against our national
interest,” he noted. ‘Singapore Issues’ refer to
the ideas championed by WTO in 1996
Singapore conference, where business
facilitation through transparency in

©www.InisightsIAS.com 11 ©www.InsightsonIndia.com
|

1. “Samudra Ratnakar” and “Sagar Nidhi” have The author of the book is
recently been seen in the news. What are they? (a) John Ruskin

(a) Names of two Scorpene-class submarines (b) Raychandbhai


being built by Mazagon Docks. (c) Gopal Krishna Gokhale

(b) Two research vessels belonging to the (d) Leo Tolstoy

Geological Survey of India and the National


Institute of Ocean Technology which have Solution: d.
been deployed for scanning the bed of Bay
Gandhi was greatly influenced by Leo Tolstoy,
of Bengal. through his book ‘The Kingdom of God is
Within You’ and his essay on ‘Christianity
(c) The two instruments that facilitated the and Patriotism’. Tolstoy’s ideal of “simplicity
discovery of natural gas hydrates in the of life and purity of purpose” influenced
Gandhi deeply. The “love as law of life” and
Bay of Bengal. principles of non-violence, that is based on
love for the entire mankind, were deeply
(d) Vessels built by the National Institute of embedded in the writings of Tolstoy. Both
Ocean Technology in collaboration with the Gandhi and Tolstoy adopted the idea of love to
solve problems of life. Gandhi has written that
Ministry of Environment, Forests and
reading the ‘The Kingdom of God is Within You’
Climate Change to conduct regular studies had cured him of skepticism and made him
on coral reefs in Indian waters. a firm believer of Ahimsa. For both of them,
non-violence could cure all social maladies,
eradicate political ills and establish peace on
earth. In summary, it was Tolstoy’s ‘The
Solution: b. Kingdom Of God Is Within You’ which made
Gandhi speak of his “inner voice” and live a life
“Two research vessels, Oceanographic vessel of “simplicity”.
Samudra Ratnakar of the Geological Survey of
India and Sagar Nidhi of the National Institute http://pib.nic.in/newsite/PrintRelease.aspx?
of Ocean Technology (NIOT), are leading the relid=149509
underwater search” for the missing An-32.
“They are capable of sonar search up to the http://www.gktoday.in/impact-of-leo-tolstoy-
depths at which the aircraft is supposed to be on-mahatma-gandhi/
and have been mapping the sea bed,” one
official said.
3. Under Phase I of the Strategic Petroleum
http://www.thehindu.com/todays-paper/tp-
national/robotic-vehicle-to-scan-sea-floor-for- Reserves storage program, three facilities have
missing-an32/article9073470.ece been created at Vishakhapatnam, Mangalore
and Padur. Which among the following are the
proposed sites for Phase II of the storage
2. Consider the following statements with
reference to the author of a book by name “The program?
Kingdom of God is Within You”:
1. Chandikhol, Odisha
1. It is said to have made Gandhiji a firm
believer in Ahimsa. 2. Bikaner, Rajasthan
2. Both, the author of the book as well as
Gandhi, believed non-violence could cure 3. Hazira, Gujarat
all social maladies and eradicate political
ills. 4. Paradip, Odisha

©www.InisightsIAS.com 12 ©www.InsightsonIndia.com
|

Select the correct answer using the code given 5. What is the predominant chemical
below:
composition of the liquid bodies on Saturn’s
(a) 1 and 2
moon Titan?
(b) 1 and 4

(c) 2 and 3 (a) Nitrogen


(d) 3 and 4
(b) Hydrocarbons

Solution: a. (c) Argon


“Under Phase II storage program, plans
currently are afoot to create Strategic (d) Water
Petroleum Reserves at two locations, namely
Chandikhol in Odisha and Bikaner in
Rajasthan which entails storage of crude oil in
underground rock caverns and underground Solution: b.
salt caverns respectively.”
Blast from the past! Based on an article which
http://pib.nic.in/newsite/PrintRelease.aspx? appeared in the Science and Technology
relid=149511 section in the Hindu’s Education Plus
Supplement last week (on August 29th, 2016).

4. Consider the following rivers: “The Titanian seas, however, are not
composed of water, like Earth’s seas, but are
1. Barak seas of liquid hydrocarbons and one of the
reasons why NASA wants to go to Titan is “to
2. Lohit determine if hydrocarbon-based life is possible
on Titan,” NASA cryogenics engineer Jason
3. Subansiri Hartwig said.

“Secondly, with clouds and an atmosphere,


Which of the above flows/flow through Titan is very similar to Earth apart from the
Arunachal Pradesh? extreme cold and oceans of liquid methane.
But hidden in the methane sea may be clues
(a) 1 only to how life evolved and potentially some weird
extraterrestrial microbes.”
(b) 2 and 3 only
Wikipedia: According to Cassini data,
scientists announced on February 13, 2008,
(c) 1 and 3 only
that Titan hosts within its polar lakes
“hundreds of times more natural gas and other
(d) 1, 2 and 3
liquid hydrocarbons than all the known oil and
natural gas reserves on Earth.”

http://indianexpress.com/article/technology
Solution: b. /science/nasa-plans-to-send-submarine-to-
CS(P), 2014 saturns-moon-titan-3000240/

http://pib.nic.in/newsite/PrintRelease.aspx?
relid=149466

©www.InisightsIAS.com 13 ©www.InsightsonIndia.com
|

1. It is due to reservations expressed by Pakistan


that the SAARC Motor Vehicle Agreement Solution: a.
(MVA) could not be signed. This consequently
“The Water Jet Fast Attack Craft (WJFAC),
led to the pursuit of a sub-regional MVA “TIHAYU” built by Garden Reach Shipbuilders
among Bangladesh, Bhutan, India and Nepal. and Engineers Limited (GRSE), Kolkata was
handed over to the Indian Navy on 30 Aug
The MVA, when operational, will regulate the 2016.” It is named after an island in the
movement of which type of vehicular traffic? Andaman.

http://pib.nic.in/newsite/PrintRelease.aspx?
(a) Passenger
relid=149533
(b) Personal

(c) Cargo 3. Which among the following South American


nations border both – the Pacific and Atlantic
(d) All of the above
Ocean?

1. Venezuela
Solution: d.
2. Ecuador
“Three countries, viz., Bangladesh, India and
Nepal have currently ratified the MVA and 3. Colombia
Bhutan is in the process of completing its
ratification formalities. To help in finalization 4. Chile
of Protocols that would facilitate trade and
maintain balance with effective controls, the Select the correct answer using the code given
countries had agreed to conduct trial runs of
vehicles, in terms of the MVA.” below:

http://pib.nic.in/newsite/PrintRelease.aspx? (a) 1 only


relid=149551
(b) 2 and 3
PIB: BBIN MVA
(c) 3 only

2. A Water Jet Fast Attack Craft (WJFAC) named (d) 3 and 4


“Tihayu” was recently handed over to the
Indian Navy. This WJFAC has been named
after Solution: d.

Colombia and Chile share their borders with


(a) An island in the Andaman. both the oceans.

(b) A tribal God worshipped by the Jarawa “The Strait of Magellan is the most important
natural passage between the Atlantic and
tribe.
Pacific oceans but it is considered a difficult
route to navigate because of the unpredictable
(c) The earliest known warship constructed by
winds and currents and the narrowness of the
the Chola empire. passage.” This strait is in possession of Chile.
See image of the tip of South America which
(d) The term used in the Mauryan empire to shows border between Chile and Argentina.
denote boats.

©www.InisightsIAS.com 14 ©www.InsightsonIndia.com
|

5. With reference to the Gravity Recovery and


Climate Experiment, consider the following
statements:

(a) It is an experiment being conducted since


2002, on the basis of measurements of
Earth’s Gravity, to study groundwater
depletion and variability across seasons in
the Ganga-Yamuna and Indus River
basins.

(b) It is a joint mission of NASA and the


German Aerospace Centre which has been
making detailed measurements of Earth’s
gravity field anomalies using satellites
launched for the same purpose.

(c) It is a study being conducted by the


Improvisation: Indian Express, “How Geological Survey of India in collaboration
Colombia’s 52-yr war came to an end”, with the United States Geological Survey to
Explained Page. understand the association of gravitation
anomalies and climate change with
hydrocarbon deposits in the oceanic crust.
4. That NASA has unveiled plans to send a
(d) None of the above.
submarine into the depths of an ocean on
Saturn’s moon Titan is well known. What is
the name of the ocean to which this submarine Solution: b.

is proposed to be deployed? Based on an article which appeared on the


30th of August, 2016.
(a) Ligeia Mare
“In recent years, several reports have warned
of alarming groundwater depletion in
(b) Punga Lacus
northwest India and Pakistan based on
satellite imagery from the Gravity Recovery
(c) Kraken Mare
and Satellite Experiment (GRACE) that has
minutely tracked how gravity varies across the
(d) Kayangan Lacuna
earth since 2002.”

 The Gravity Recovery And Climate


Experiment (GRACE), a joint mission of
Solution: c.
NASA and the German Aerospace Center,
Based on an article which appeared in the has been making detailed measurements
Science and Technology section in the Hindu of Earth’s gravity field anomalies since its
on August 29th, 2016. launch in March 2002.
 Gravity is determined by mass. By
“NASA is working on sending a submarine into measuring gravity anomalies, GRACE
the depths of the Kraken Mare.” shows how mass is distributed around the
Kraken Mare is the largest ocean on Titan. planet and how it varies over time.
 Data from the GRACE satellites is an
http://indianexpress.com/article/technology important tool for studying Earth’s ocean,
/science/nasa-plans-to-send-submarine-to- geology, and climate.
saturns-moon-titan-3000240/
Improvisation:
http://www.thehindu.com/data/water-table-
rises-in-ganga-basin-but-so-does-
salinity/article9051315.ece

©www.InisightsIAS.com 15 ©www.InsightsonIndia.com
|

1. In the Mekong-Ganga Cooperation, an Solution: d.


initiative of six countries, which of the
following is/are not a participant/ “Barely six weeks after participating in the G-
participants? 20 summit at Hangzhou, China, and the East
Asia and ASEAN-India summits at Vientiane,
1. Bangladesh Laos, Prime Minister Narendra Modi will
himself play host to the annual BRICS
2. Cambodia summit in Goa on October 15-16, 2016. This
3. China will bring together the heads of state of Brazil,
Russia, India, China and South Africa, and the
4. Myanmar summit theme put forward by India is
“Building Responsive, Inclusive and Collective
5. Thailand
Solutions”, which is a clever play on the letters
Select the correct answer using the code given constituting the membership of the grouping.”
below.
http://www.thehindu.com/opinion/lead/on-
(a) 1 only indias-diplomacy-at-the-brics-summit-in-
goa/article9115819.ece
(b) 2, 3 and 4

(c) 1 and 3
3. The host of a BRICS Summit has the privilege
(d) 1, 2 and 5 to arrange a regional outreach. For instance,
Russia had invited the leaders of the Eurasian
Economic Union and the Shanghai
Solution: c. Cooperation Organisation at Ufa last year.
India, being the host in 2016, has chosen to
CS(P), 2015; invite the leaders of
“The Mekong–Ganga Cooperation (MGC) was (a) South Asian Association for Regional
established on November 10, 2000 at Cooperation
Vientiane at the First MGC Ministerial
Meeting. It comprises six member countries, (b) Indian Ocean Rim Association
namely India, Thailand, Myanmar,
(c) Bay of Bengal Initiative for Multi-Sectoral
Cambodia, Laos and Vietnam. They
Technical and Economic Cooperation
emphasised four areas of cooperation, which
are tourism, culture, education, and (d) Mekong-Ganga Cooperation
transportation linkage in order to be solid
foundation for future trade and investment
cooperation in the region.”
Solution: c.
http://www.thehindu.com/opinion/lead/on-
“At the Fortaleza summit in 2014, Brazil had
indias-diplomacy-at-the-brics-summit-in-
invited several heads of state/government
goa/article9115819.ece
from Latin America, while at Ufa last year
Russia had invited the leaders of the Eurasian
Economic Union and the Shanghai
2. ‘Building Responsive, Inclusive and Collective Cooperation Organisation. One would have
Solutions’ is the theme for which among the expected India to have invited SAARC leaders
following summits being held this year? to the outreach but it has chosen to host the
leaders of the seven-member BIMSTEC (Bay of
(a) ASEAN-India Summit, Laos
Bengal Initiative for Multi-Sectoral Technical
(b) SAARC Summit, Pakistan and Economic Cooperation) instead. BIMSTEC
is a potential Bay of Bengal Economic
(c) G20 Summit, China Community comprising Bhutan, Bangladesh,
(d) BRICS Summit, India India, Nepal and Sri Lanka from South Asia
and Myanmar and Thailand from ASEAN.”

©www.InisightsIAS.com 16 ©www.InsightsonIndia.com
|

http://www.thehindu.com/opinion/lead/on-
indias-diplomacy-at-the-brics-summit-in-
goa/article9115819.ece NOTES

4. “Oru jati, oru matam, oru daivam


manushyanu”, or, “One caste, one religion,
one God for humankind” is a statement
ascribed to

(a) C. N. Annadurai

(b) E. V. Ramaswamy Naicker

(c) Shri Narayana Guru

(d) Tulsidas

Solution: c.

“Shri Narayana Guru, proclaimed the ideals of


unity for his people. He argued against treating
people unequally on the basis of caste
differences. According to him, all humankind
belonged to the same caste.”

Page 116, Our Pasts – III Part 2, NCERT Class


8

http://pib.nic.in/newsite/PrintRelease.aspx?
relid=149829

5. NASA’s ‘Cassini’ Spacecraft recently entered


the final year of its epic voyage during which it
will make the closest-ever observations of the
planet

(a) Jupiter

(b) Saturn

(c) Mars

(d) Uranus

Solution: b.

“After studying Saturn, its rings and moons for


more than 12 years, NASA’s Cassini spacecraft
has entered the final year of its epic voyage
during which it will make the closest-ever
observations of the planet.”

http://www.thehindu.com/todays-paper/tp-
national/cassini-to-make-final-observation-
of-saturn/article9116758.ece

©www.InisightsIAS.com 17 ©www.InsightsonIndia.com
|

1. Construction and launch (first contact with indias-latest-missile-


water) of ‘Mormugao’, the second ship of destroyer/article9118937.ece?photo=1
Project 15B, was recently completed. With
reference to Project 15B, which of the following http://pib.nic.in/newsite/PrintRelease.aspx?
statements is/are incorrect? relid=150861

1. Ships being constructed under this project


belong to the Visakhapatnam-class Guided 2. The Supreme Court, in a recent case decided
Missile Destroyers. to opt for a “judicial innovation” instead of the
2. These ships are based on the older Project death penalty. This “judicial innovation” which
15A Kolkata Class destroyers. was formalised by a Constitution Bench of the
SC in the Rajiv Gandhi killers’ case recently,
3. The gas engine turbines driving these implies which of the following?
ships have been designed and constructed
indigenously. (a) The convict is awarded rigorous life
imprisonment (instead of the death
Select the correct answer using the code given penalty) till natural death.
below:
(b) It substitutes the death penalty with a
(a) 1 and 2 “special category” of life imprisonment
without the benefit of release on remission
(b) 2 only for prolonged periods ranging from 25 to
(c) 1 and 3 only 30 years.

(d) 3 only (c) The convict is awarded the death penalty


but in reality is allowed to live out his/her
life till natural death, without a chance to
apply for remission.
Solution: d.
(d) It substitutes the death penalty with
The Diplomat: “The warships of the
solitary confinement for the entire
Visakhapatnam-class are based on the older
duration of life imprisonment awarded.
Project 15A Kolkata-class destroyers design,
three of which will join the Indian Navy by the
end of 2016. The Visakhapatnam-class is the
most advanced destroyer class ever to be built Solution: b.
in India, according to Indian defense officials. “This judicial innovation, formalised by a
Displacing 7,300 tons, the stealth guided Constitution Bench of the Supreme Court in
missile destroyer has a crew of 50 officers and the Rajiv Gandhi killers’ case in December
250 sailors. It is powered by four Ukrainian 2015, helps “get rid of death penalty” and
gas turbine engines, and capable of reaching a addresses the genuine concerns of the society
maximum speed of about 30 knots. The ship’s to see justice done, a three-judge Bench led by
operational range is around 4,000 nautical Justice J. Chelameswar observed in its recent
miles.” judgment. The innovation involves
“Around 65 percent of components of the substituting death penalty with a “special
Vishakhapatnam-class are domestically category” of life imprisonment without the
designed and produced, including 11 of its benefit of release on remission for prolonged
weapon and associated sensor systems, periods ranging from 25 to 30 years, if not
according to Indian naval officials. The more.
principal armament of the Visakhapatnam- This innovative approach veering away from
class will be eight BrahMos supersonic cruise capital punishment was formalised after the
missiles” and it “will be equipped with the Supreme Court gave itself the authority to
Barak-8/NG — an Indo-Israeli surface-to-air tweak the sentencing laws and evolve a special
missile (SAM)”. category of sentence in its judgment in Union
http://www.thehindu.com/news/national/si of India versus Sriharan alias Murugan last
x-things-you-need-to-know-about-mormugao- year. The special category is to be limited to a

©www.InisightsIAS.com 18 ©www.InsightsonIndia.com
|

“very few cases”. This special category finds its copyright infringement petition initiated in
first mention in the Swami Shraddananda August 2012 by three publishers (Oxford,
versus State of Karnataka judgment of the Cambridge and Taylor & Francis) against a
Supreme Court in 2008. photocopy shop located in the premises of
Delhi University”
The innovation, according to Justice S.K.
Singh, who authored the Lodhi judgment for http://www.thehindu.com/opinion/lead/jud
Justice Chelameswar’s Bench, is an gment-in-the-delhi-university-photocopying-
endeavour by the apex court to make “no party case-a-blow-for-the-right-to-
(convict or the society) a loser”. So having knowledge/article9121260.ece
saved Lodhi from the hangman’s rope, Justice
Singh stripped him of his right to apply for
release from prison on remission for the next 4. Which of the following statements with
25 years. Thus, any hope Lodhi might have reference to the Non-Aligned Movement is/are
had for his release after serving the first 14 incorrect?
years was effectively extinguished.”
1. The movement was officially founded in
http://www.thehindu.com/news/national/ju Belgrade in 1961.
dicial-innovation-helps-supreme-court-avoid-
awarding-death-penalty/article9121307.ece 2. The chairmanship rotates between
countries and changes at every summit of
heads of state or government to the
3. Section 52(1) of the Copyright Act, 1957 deals country organising the summit.
with 3. The chair is at present occupied by Iran.
(a) The duration of copyright allotted to Select the correct answer using the code given
different works – literary, dramatic, below:
musical and artistic works.
(a) 1 and 2 only
(b) The remedies available to owners of
copyright for instances of its infringement. (b) 2 and 3 only

(c) Exceptions to what constitutes ‘copyright (c) 3 only


infringement’.
(d) None of the above.
(d) The rights in India of foreign creators and
owners of copyrights.
Solution: c.

The 16th Summit was held in Iran in 2012. It


Solution: c.
assumed the chair then from Egypt, and the
Section 52(1) in the Copyright Act, 1957 latest summit – the 17th NAM Summit –
recently concluded in Venezuela, has resulted
Relevant for our purpose: Section 52(1)(i) – The in Venezuela occupying the chair.
following acts shall not constitute an
infringement of copyright, namely: “I congratulate the Bolivarian Republic of
Venezuela for assuming the Chairmanship of
 by a teacher or a pupil in the course of the Non-Aligned Movement. I felicitate His
instruction; or Excellency President Hassan Rouhani for the
 as part of the questions to be answered in outstanding leadership that the Islamic
an examination; or Republic of Iran provided to our Movement
 in answers, to such questions; over the last four years.”
“In its much awaited judgment in the Delhi http://pib.nic.in/newsite/PrintRelease.aspx?
University photocopying case (The Chancellor relid=150874
Masters and Scholars of the University of
Oxford v. Rameshwari Photocopy Services),
the Delhi High Court has dismissed the

©www.InisightsIAS.com 19 ©www.InsightsonIndia.com
|

https://en.m.wikipedia.org/wiki/Non- http://pib.nic.in/newsite/PrintRelease.aspx?
Aligned_Movement#Organizational_structure_ relid=150874
and_membership

6. ‘Global Fishing Watch’, recently launched, is


5. The theme for the Non-Aligned Movement for
the next three years is (a) An Organisation, established under a Food
and Agriculture Organisation convention,
(a) Peace, Sovereignty and Solidarity for comprising governments of major-fishing
Development. nations and Non-Governmental
Organisations working towards promoting
(b) Lasting Peace through Joint Global sustainable fishing practices.
Governance.
(b) It is a technology-tool, dependent on
(c) Cooperation and Joint Progress towards access to the Internet, which aims to offer
Sustainable Development Goals. a crowd-sourced solution to the problem of
(d) World Peace for Sustainable Development. illegal fishing.

(c) A purely Non-Governmental Organisation


which will provide for technical, financial
Solution: a. and educational support for the proper
conservation of fish species whose
“Our theme for the next three years – “Peace,
existence are under threat due to illegal
Sovereignty and Solidarity for Development” –
fishing.
is in congruence with our founding principles.”
As a Movement that has stood steadfastly in (d) An organisation under the Food and
the forefront by highlighting global economic Agriculture Organisation which originally
issues that affect the developing world, we dealt with illegal, unreported and
must ensure wholehearted and holistic unregulated fishing in the Asia-Pacific
implementation of the SDGs and remain region but now has been revamped to cover
attentive to any effort to modify or distort the same functions across all seas and
Agenda 2030 by laying overemphasis on oceans.
particular SDGs over others, or by trying to
create a standard of measurement that could
be intrusive and has no relationship to Solution: b.
national contexts.
“American movie star Leonardo DiCaprio
Sustainable development, Excellency, is both unveiled on Thursday, a new, free technology
the bedrock and the highest aspiration of our that allows users to spy on global fishing
efforts; hence the importance of solidarity that practices, in a bid to curb illegal fishing.”
this Movement attaches to achieving it. This is
epitomised in the theme of the Summit –  “It aims to offer a crowd sourced solution
Solidarity for Development. to the problem of illegal fishing, which
accounts for up to 35 per cent of the global
The other two elements of our triad of wild marine catch and causes yearly losses
objectives – Peace and Sovereignty – are a pre- of $23.5 billion, according to the UN Food
requisite for development. A peaceful global and Agriculture Organization (FAO).”
environment is essential for development and  “Using satellite technology combined with
for development cooperation; such cooperation radar aboard boats, the site
must be undertaken in exercise of the globalfishingwatch.org allows people to
sovereign will of each member of the zero in on areas of interest around the
community of nations. The interdependence of world and trace the paths of 35,000
the three is thus evident and essential. The commercial fishing vessels.”
impediments to each must also be discerned
clearly.” http://www.thehindu.com/sci-tech/energy-
and-environment/leonardo-dicaprio-unveils-

©www.InisightsIAS.com 20 ©www.InsightsonIndia.com
|

free-technology-to-spy-on-illegal- 9. Consider the following pairs:


fishing/article9115109.ece
River States involved in
dispute over water-
sharing
7. “That which bends up” is the meaning for the
common name of a disease. Which disease is 1. Mahadayi : Karnataka and Goa
being referred to here?
2. Kaveri : Karnataka and Tamil
(a) Dengue Nadu

(b) Chikungunya 3. Mahanadi : Odisha and Chattisgarh

(c) Hansen’s Disease Which of the pairs above is/are not correctly
(d) Leprosy matched?

(a) 1 only

Solution: b. (b) 2 and 3 only

Hansen’s Disease is Leprosy. Chikungunya (c) 3 only


has the effect of causing severe joint pain, (d) None of the above
somewhat similar to arthritis, causing the
curling up of fingers and hands and so on –
“that which bends up”.
Solution: d.
“… “That which bends up” is what the word
All pairs are correctly matched.
“Chikungunya” means in Kimakonde, an
African language. It’s quite an apt word Modified question; original question was
considering the effect this infection has on posted on August 25th.
those affected by it! …”
http://www.thehindu.com/news/national/ot
http://www.thehindu.com/opinion/op- her-states/no-decision-on-odishas-next-
ed/chikungunya-an-uneasy- move/article9122200.ece
buzz/article9119210.ece

10. Which country recently permitted for the first


8. It was after three years that surplus waters time a minor to be euthanised, after its rules
were recently released from the Musi Dam. were adopted in 2014 allowing doctor-assisted
River Musi is a tributary of River death for minors of all ages?
(a) Baitarani (a) Canada
(b) Krishna (b) Belarus
(c) Mahanadi (c) Belgium
(d) Cauvery (d) Netherlands

Solution: b. Solution: c.
River Musi is a tributary of River Krishna. “Belgium legalised euthanasia in 2002, and
two years ago amended the rules to permit
http://www.thehindu.com/todays-paper/tp-
doctor-assisted death for minors. It is the only
national/foam-and-fury/article9119746.ece
country in the world allowing euthanasia on
http://www.irrigation.telangana.gov.in/icad/ minors of all ages. In the neighbouring
static/projects/musi.html Netherlands the practice is legal for children
aged at least 12. The Belgian law allows

©www.InisightsIAS.com 21 ©www.InsightsonIndia.com
|

euthanasia for minors under strict conditions.


Minors have to be conscious and able to make
rational decisions when they request assisted NOTES
death. Psychologists or psychiatrists must be
consulted and the authorisation of the parents
is required. To undergo euthanasia, the minor
must also be in a “terminal medical situation
with constant and unbearable physical pain
which cannot be assuaged and that will cause
death in the short term,” the law says.

http://www.thehindu.com/news/internation
al/first-mercy-killing-of-minor-in-
belgium/article9118814.ece

11. Consider the following statements:

1. The name means “worshippers of God”,


but the community has often been referred
to as “devil worshippers”.

2. Their own name for themselves is


“Daasin”.

3. The revere both the Bible and the Koran.

The above statements refer to which of the


following communities recently seen in the
news?

(a) Basee

(b) Taha

(c) Murad

(d) Yazidi

Solution: d.

“A young Iraqi woman, who survived


trafficking at the hands of the Islamic State
(IS), has been appointed a United Nations
Goodwill Ambassador for the dignity of
survivors of human trafficking. Nadia Murad
Basee Taha (23), a Nobel Peace Prize nominee,
is the Goodwill Ambassador for the Dignity of
Survivors of Human Trafficking of the UN
Office on Drugs and Crime.”

http://www.thehindu.com/todays-paper/tp-
international/trafficking-survivor-from-yazidi-
community-made-un-
ambassador/article9119696.ece

http://www.bbc.com/news/blogs-magazine-
monitor-28686607

©www.InisightsIAS.com 22 ©www.InsightsonIndia.com
|

1. It was recently found that the genetic Which of the statements given above is/are
differences between an animal species ‘X’ was correct?
so large that it had to be split up into four new
(a) 1 only
species. They have been named, Southern ‘X’,
the Masai ‘X’, Reticulated ‘X’ and Northern ‘X’. (b) 2 only
‘X’ refers to
(c) Both 1 and 2
(a) Zebra
(d) Neither 1 nor 2
(b) Black Mamba

(c) Caracal Solution: a.


(d) Giraffe “The first catalogue of more than a billion stars
from ESA’s Gaia satellite was published
today – the largest all-sky survey of celestial
Solution: d. objects to date. On its way to assembling the
most detailed 3D map ever made of our Milky
“Genetic research on the world’s tallest land Way galaxy, Gaia has pinned down the precise
animal has found that there are four distinct position on the sky and the brightness of 1142
species of giraffe, not just one as long million stars.”
believed, with two of them at alarmingly low
population levels.” “The European Space Agency (Esa) launched
its Gaia mission in 2013. Its goal was to
“Giraffes have previously been recognised to be update and extend the work of a previous
a single species divided into several sub- satellite from the 1980s/90s called Hipparcos:
species. But this latest study of their DNA This observatory made the go-to Milky Way
suggests that four groups of giraffes have not catalogue for its time – an astonishing chart of
cross-bred and exchanged genetic material for our cosmic neighbourhood. It mapped the
millions of years. This is a clear indication that precise position, brightness, distance and
they have evolved into distinct species. The proper motion (that sideways movement on the
scientists say their findings could inform the sky) of 100,000 stars. Gaia, with its first
conservation efforts for all four species of release of data, has just increased that haul
giraffe.” 20-fold.”
September 16th, Explained Page, Indian September 16th, Explained Page, Indian
Express or Reuters Express
http://www.thehindu.com/sci- http://m.esa.int/Our_Activities/Space_Scien
tech/science/giraffes-are-four-species-not- ce/Gaia/Gaia_s_billion-
one-study/article9090867.ece star_map_hints_at_treasures_to_come
http://www.bbc.com/news/science-
environment-37311716
3. ‘Tiangong 2’ is China’s

(a) Second manned flight.


2. Consider the following statements with
reference to ‘Gaia’, which was recently in the (b) Second Moon-rover mission.
news:
(c) First experimental space station.
1. It is a space telescope launched by the
(d) None of the above
European Space Agency.

2. It recently mapped over a billion stars in


the Milky Way as well as the Andromeda Solution: d.
Galaxy.
It is China’s second experimental space station.

©www.InisightsIAS.com 23 ©www.InsightsonIndia.com
|

“The mission follows the launch of the Page 85, Economic Survey 2015-16, Volume 2
Tiangong 1 prototype in 2011, a smaller but
also operational model. The Tiangong 2 is still http://www.thehindu.com/todays-paper/tp-
an operational space station, even if it is still business/nri-deposits-drive-up-external-
only a prototype ahead of the real thing. Once debt-to-486-bn/article9125696.ece
the crew arrive next month they will embark
on research projects. Quantum
communications, gamma ray burst research 5. The State shall endeavour to secure for the
and fluid physics. There is also the (slightly citizens a uniform civil code throughout the
easier to grasp) research on plant growth in territory of India” – which article in the Indian
space.” Constitution places this duty upon the State?

http://www.bbc.com/news/world-asia- (a) Article 43


china-37370278
(b) Article 44
http://www.thehindu.com/sci-
(c) Article 45
tech/science/china-launches-second-
experimental-space-lab- (d) Article 46
module/article9111553.ece

Solution: b.
4. Consider the following statements with
reference to India’s external debt: http://indiankanoon.org/doc/1406604/

1. The maturity pattern of India’s external http://www.thehindu.com/opinion/lead/mar


debt shows the predominance of long-term y-e-john-hasina-khan-on-lives-of-muslim-
borrowings. women/article9124987.ece

2. Its single largest component is Non-


Resident Indian deposits.
6. Which of the following government entities is
Which of the statements given above is/are associated with building Supercomputers?
correct?
(a) SAMEER
(a) 1 only
(b) C-DAC
(b) 2 Only
(c) CCA
(c) Both 1 and 2
(d) NIC
(d) Neither 1 nor 2

Solution: b.
Solution: a.
“Union Human Resource Development
“The composition of external debt also reflects Minister Shri Praksh Javadekar launched
a well-maintained longer maturity profile and PARAM-ISHAN supercomputing facility at
broad balance in terms of sources. The IIT, Guwahati on 19th September, 2016. ”
increase in external debt at end-March 2015
“Centre for Development of Advanced
was due to higher levels of commercial
Computing (C-DAC) is the premier R&D
borrowings, particularly commercial bank
organization of the Ministry of Electronics and
loans and securitized borrowings, and NRI
Information Technology (MeitY) for carrying out
deposits. ECBs have been a crucial
R&D in IT, Electronics and associated areas.”
determinant of the magnitude of India’s
external debt and its single largest “The setting up of C-DAC in 1988 itself was to
component.” built Supercomputers in context of denial of
import of Supercomputers by USA. Since then
NRI deposits are classified as long-term
C-DAC has been undertaking building of
external debt.

©www.InisightsIAS.com 24 ©www.InsightsonIndia.com
|

multiple generations of Supercomputer 8. “Yatri Mitra Sewa” was recently introduced


starting from PARAM with 1 GF in 1988.”
(a) By the Ministry of Culture and Tourism
http://cdac.in/index.aspx?id=CorporateProfil with a view to providing fair and
e reasonably-priced services to foreign
tourists by individuals trained specifically
http://pib.nic.in/newsite/PrintRelease.aspx? for the purpose.
relid=150905
(b) By the Ministry of Railways to enable
passengers – both differently-abled as well
7. Consider the following statements: as old persons – to book wheelchair
services cum porter services at railway
1. Kushan : Seated Buddha stations.
period
(c) By the Ministry of Railways to enable only
2. Chola dynasty : Goddess Pratyangira
differently-abled passengers to book
wheelchair services cum porter services at
3. Stupa of : Worshippers of the railway stations.
Chandavaram Buddha
(d) By the Ministry of Culture and Tourism to
make easier and convenient and
The above mentioned sculptures were in the
consequently promote intra-country
news recently for which among the following
tourism by stationing at every state
reasons?
capital’s railway station a certain number
(a) These ancient artefacts were recently of individuals who together can speak
excavated. every official language in Schedule 8 of the
Constitution.
(b) The first set of ancient artefacts to be
handed over to the Archeological Survey of
India under the Voluntary Deaccession
Solution: b.
Scheme.
 “Yatri Mitra shall provide Wheel chair cum
(c) The National Gallery of Australia handed
porter services to differently abled,
them over to India.
ailing and old persons.”
(d) These artefacts were stolen while they were  “A Yatri Mitra can either be a Sahayak or
being transported for the purpose of any other person provided by IRCTC or the
restoration. service provider appointed by IRCTC for
this purpose.”
 “The responsibility of providing Yatri Mitra
Solution: c. Sewa has been entrusted with IRCTC.
IRCTC may provide this service ‘Free of
“Dr. Mahesh Sharma, Minister of State (I/C) cost’ through some NGO, charitable trust,
for Culture & Tourism attended a special event PSUs etc under CSR. However, if this
at National Gallery of Australia (NGA) in service can’t be provided ‘Free of Cost’ due
Canberra, Australia today in which Senator to lack of response from NGOs, Charitable
Mitch Fifield formally handed over three trust, PSUs etc, IRCTC may arrange this
ancient artefacts stolen and smuggled out of service on payment basis through a service
India and inadvertently acquired by National provider or on its own.”
Gallery of Australia. The returned artefacts are
Seated Buddha, Goddess Pratyangira and http://pib.nic.in/newsite/PrintRelease.aspx?
Worshippers of Buddha.” relid=150899

http://pib.nic.in/newsite/PrintRelease.aspx?
relid=150906

http://pibphoto.nic.in/documents/rlink/201
6/sep/p201691901.pdf

©www.InisightsIAS.com 25 ©www.InsightsonIndia.com
|

1. ‘SWAYAM’ Programme, sometimes seen in the 2. With reference to Barak-8, which was
news, may refer to successfully test fired recently, consider the
following statements:
(a) An Information Technology platform
hosting Open Online Courses offered by 1. It has been developed jointly by Israel and
Professors of centrally funded institutions, India.
being run by the Ministry of Human
2. It is a surface-to-air, nuclear-capable
Resource Development.
ballistic missile.
(b) An initiative by the Ministry of Micro, Small
3. It is designed to defend against any
and Medium Enterprises to provide
airborne threat including supersonic
vocational training to school dropouts to
aircrafts and missiles.
improve their employability.
Which of the statements above is/are correct?
(c) A programme initiated by the Ministry of
Finance in collaboration with the Ministry (a) 1 only
of Women and Child Development to
(b) 2 and 3 only
provide access to low-cost finance to
widows and victims of trafficking who wish (c) 1 and 3 only

to become self-employed. (d) 1, 2 and 3

(d) All of the above.

Solution: d.

Solution: a. “India Tuesday successfully test-fired the


Barak-8 long-range surface-to-air nuclear-
“IIMs will actively participate in creating capable ballistic missile, jointly developed
contents for SWAYAM which is an Information with Israel, from a defence test facility off the
Technology platform hosting Open Online Odisha coast.
Courses and will provide high quality
education on various subjects from 9 to Under  The advanced missile has been designed
Graduate and Post Graduate students — and developed by the DRDO and Israel
covering all disciplines. SWAYAM is a free Aerospace Industries and Israel’s
course which can be availed online and shall Administration for the Development of
be a complete channel of new Open Learning Weapons and Technological
system comprising of lectures, reading Infrastructure.
materials, tutorials, mid exams, final exams  The Long-Range Surface-to-Air Missile
and certification.” (LR-SAM) has the ability to hit targets
within radii of 70 km to 90 km.
http://pib.nic.in/newsite/PrintRelease.aspx?
 It is designed to defend against any
relid=150946
airborne threat, apart from aircraft and
http://vikaspedia.in/education/policies-and- helicopters, and can also intercept
schemes/new-initiatives-taken-by-the- supersonic aircraft and missiles.
ministry-of-hrd#section-13  Barak-8 is based on the original Barak-1
missile but has a more advanced target-
seeker. The radar system provides 360-
degree coverage and the missile can take
down an incoming missile as close as 500
meters away from a ship.”

©www.InisightsIAS.com 26 ©www.InsightsonIndia.com
|

http://www.business- https://en.m.wikipedia.org/wiki/Kamakhya_
standard.com/article/news-ians/india- Temple
successfully-test-fires-barak-8-missile-
116092001021_1.html Improvisation:
http://pib.nic.in/newsite/PrintRelease.aspx?
http://www.thehindu.com/news/national/in relid=150945
dia-sucessfully-testfires-surfacetoair-
missile/article9127725.ece
4. Consider the following statements:

3. With reference to the Kamakhya Temple in 1. The Montreal Protocol deals only with
Assam, which of the following statements phasing down of substances that deplete
is/are incorrect? the ozone layer.

1. It is a ‘Shakti Peeth’. 2. Unlike the Montreal Protocol, in which


each of the signatories is equally
2. It is the main temple within a complex of responsible for eliminating the banned
individual temples dedicated to the ten chemicals, the Kyoto Protocol puts
Mahavidyas. “differentiated responsibilities” on
developed and developing countries.
3. It is an important pilgrimage for Tantric
worshippers. Which of the statements given above is/are
correct?
Select the correct answer using the code given
below: (a) 1 only

(a) 1 and 2 only (b) 2 only

(b) 2 and 3 only (c) Both 1 and 2

(c) 1 and 3 only (d) Neither 1 nor 2

(d) None of the above

Solution: b.

Solution: d. “The Kyoto Protocol is an international treaty


that commits State Parties to reduce
 “The Shakti Peetha is a place of worship greenhouse gas emissions, based on the
consecrated ashes of the goddess Shakti or premise that (a) global warming exists and (b)
Sati, the female principal of Hinduism and human-made CO2 emissions have caused it.
the main deity of the Shakta sect.” The Protocol is based on the principle of
 “Shaktism or Shaktidharma (“doctrine of common but differentiated responsibilities.”
power” or “doctrine of the Goddess”) is a
category of tantric Saivism, where various US EPA: “On November 6, 2015, all 197 parties
goddesses are viewed as the central deity to the Montreal Protocol on Substances that
of their respective systems.” Deplete the Ozone Layer agreed on a “Dubai
 “Mahavidyas (Great Wisdoms) or Dasha- Pathway” for controlling climate-change-
Mahavidyas are a group of ten aspects of inducing hydrofluorocarbons (HFCs). Like the
Adi Parashakti in Hinduism. After the ozone-depleting substances (ODS) they were
decline of Buddhism in India, Sakta and designed to replace, most hydrofluorocarbons
Buddhist goddesses were combined to (HFCs) are potent greenhouse gases (and HFCs
form this list of ten.” have zero Ozone-Depleting potential).
 “The development of Mahavidyas Emissions of HFCs are projected to increase
represents an important turning point in nearly twentyfold in the coming decades. This
the history of Shaktism as it marks the rise projected increase in HFC emissions would
of Bhakti aspect in Shaktism, which offset many of the climate benefits achieved by
reached its zenith in 1700 CE.” phasing out ODS under the Montreal Protocol
on Substances that Deplete the Ozone Layer.”

©www.InisightsIAS.com 27 ©www.InsightsonIndia.com
|

India in 2015:“In line with an assurance it As part of the project’s developmental work,
gave to the United States last year to work state highways and roads would be
towards reduction of hydrofluorocarbons constructed apart from establishing water
(HFCs), India has proposed a draft amendment treatment plants, water pipes, storm drains,
to the Montreal Protocol to enable a complete power substations and power distribution
phase-out of the HFCs in a time-bound lines.
manner.”
http://www.thehindu.com/todays-paper/tp-
http://www.thehindu.com/opinion/columns business/coastal-corridor-adb-approves-631-
/will-the-paris-pact-succeed-like-the- mn-loan/article9129564.ece
montreal-protocol/article9128764.ece

Notes
5. The Asian Development Bank (ADB) recently
approved a loan for construction of India’s first
Coastal Industrial Corridor. This Coastal
Industrial Corridor will be developed between

(a) Visakhapatnam and Chennai

(b) Kolkata and Tuticorin

(c) Mumbai and Kozhikode

(d) Kandla and Thiruvananthapuram

Solution: a.

“The Asian Development Bank (ADB) approved


to dole out $631 million for developing India’s
first coastal industrial corridor along the
Vishakapatnam-Chennai coastline.”

This is the first phase of development in the


2,500-kilometre-long East Coast Economic
Corridor project that is being considered
crucial in spurring development in India’s
eastern coast and establishing seamless trade
links with South and Southeast Asia.

The Visakhapatnam-Chennai Industrial


Corridor section of the East Coast Economic
Corridor, connecting four economic hubs and
nine industrial clusters, will mark the first
industrial corridor developed along India’s
coast.

The East Coast Economic Corridor will


ultimately extend from Kolkata in West
Bengal in the northeast of India to Tuticorin
in Tamil Nadu near the southern-most point of
the country.

©www.InisightsIAS.com 28 ©www.InsightsonIndia.com
|

1. Consider the following statements: Select the correct answer using the code given
below:
1. A lower figure of Operating Ratio is
indicative of better financial health of a (a) 1 only
system.
(b) 2 and 3 only
2. The Operating Ratio for the Indian
Railways is at present greater than 1. (c) 3 only

Which of the statements given above is/are (d) None of the above
correct?

(a) 1 Only Solution: d.


(b) 2 Only All three are among the several
(c) Both 1 and 2 recommendations made by the committee.

(d) Neither 1 nor 2 “The committee has recommended it not as a


stand-alone step, but as part of a slew of
measures such as: complete overhaul of the
project financing architecture of the Railways
Solution: a. involving ruthless weeding out of
“The operating ratio is a financial term unviable/long-pending projects;
defined as a company’s operating expenses as comprehensive accounting reforms;
a percentage of revenue.” The OR of the separation of infrastructure and operations;
Railways stood at 91.3% (0.913) in 2014-15. and setting up of a rail regulatory authority.
Pending these steps, each of which is a major
Page 45, Economic Survey, 2015-16 Volume 2 project in itself (some politically sensitive), the
move to give a hasty send-off to the Railway
Improvisation:
Budget is perplexing.”
http://www.thehindu.com/opinion/op-
ed/railway-budget-a-vanishing-trick-writes-k- http://www.thehindu.com/opinion/op-
balakesari/article9132602.ece ed/railway-budget-a-vanishing-trick-writes-k-
balakesari/article9132602.ece

2. Which among the following is/are not


recommendations made by the Bibek Debroy 3. Consider the following statements:

Committee for “Mobilisation Of Resources For 1. Aquaculture refers to the breeding,


rearing, and harvesting of animals as well
Major Railway Projects And Restructuring Of
as plants in all types of water
Railway Ministry And Railway Board”? environments.

1. The Indian Railways (IR) should move to 2. The Marine Products Export Development
Authority is the fisheries sector
standard commercial accounting.
coordinating agency under the Ministry of
Commerce.
2. IR schools must be integrated into the
Kendriya Vidyalaya Sangathana set-up. Which of the statements given above is/are
correct?
3. Private sector should be allowed into (a) 1 only
running freight as well passenger trains in
(b) 2 only
competition with the IR.
(c) Both 1 and 2

(d) Neither 1 nor 2

©www.InisightsIAS.com 29 ©www.InsightsonIndia.com
|

Solution: c. “With increase in discount to 80 per cent, it is


expected that the Shipping Service Providers
“Aquaculture – also known as fish or shellfish will be able to attract more auto-mobile cargo
farming — refers to the breeding, rearing, and through the coastal route and decongest the
harvesting of plants and animals in all types of already congested roads and railways and also
water environments including ponds, rivers, make the Ro-Ro ship service operations more
lakes, and the ocean.” sustainable.”
“The USA and South East Asia are the major http://pib.nic.in/newsite/PrintRelease.aspx?
importers of Indian seafood and frozen relid=150989
shrimp continued to be the major export item
last year, followed by frozen fish. Small and
marginal farmers in India, who contribute to
the bulk of coastal aquaculture are organising 5. The ‘Prakash Singh’ case, or the ‘Prakash
to stave off competition from countries such as Singh vs Union of India’ case, is well-known for
Thailand and Vietnam by boosting production its judgment on
and adopting global standard marketing (a) Guidelines to deal with sexual harassment
strategies such as certification, traceability at the workplace.
and eco-labelling.”
(b) Police reforms.
http://pib.nic.in/newsite/PrintRelease.aspx?
relid=150992 (c) Right to Hawk.

(d) Investigation of armed forces for excesses


committed in the course of discharge of
4. The Ministry of Shipping recently decided that their duty even in “disturbed areas”.
all Major Ports will provide discount of 80 per
cent for two years on Vessel Related Charges
(VRC) & Coastal Related Charges (CRC) for
Solution: b.
“Ro-Ro” ships. These ships most appropriately
refer to “The 10th anniversary of the Supreme
Court’s judgement in the Prakash Singh case
(a) Vessels designed to carry wheeled cargo,
should be one of them — a reason to look back
such as cars and trucks.
with pride at the court’s seven directions in
(b) Large trawlers that have proliferated in its September 22, 2006, verdict aimed at
recent times due to the increasing demand propelling police reform. The judgement was
for marine resources, but whose growth intended — but perhaps not expected — to
has been restricted owing to costs kick-start police reform.”
associated with docking at Major Ports.
http://indianexpress.com/article/opinion/co
(c) Vessels designed and built in India. lumns/model-police-law-supreme-court-
prakash-singh-case-india-police-3043093/
(d) Vessels designed to carry twenty-foot
shipping containers.
6. “Wolbachia” may refer to

Solution: a. (a) The key ingredient in the solution being


sprayed to rid the coal town Jharia of its
“Roll-on/roll-off (RORO or ro-ro) ships are hundred-year-old coal fires.
vessels designed to carry wheeled cargo, such
as cars, trucks, semi-trailer trucks, trailers, (b) A bear species believed now to be extinct,
and railroad cars, that are driven on and off the first of its kind due to human-induced
the ship on their own wheels or using a climate change.
platform vehicle, such as a self-propelled
(c) Mumbai and Kozhikode
modular transporter. This is in contrast to lift-
on/lift-off (LoLo) vessels, which use a crane to (d) The name given to the earliest group of
load and unload cargo.” humans to have migrated from Africa.

©www.InisightsIAS.com 30 ©www.InsightsonIndia.com
|

including: (i) the principle of common but


Solution: c. differentiated responsibilities;”

“India also plans to hold trials with mosquitoes


infected with bacteria that suppress dengue The answer once again changes! Statement 2
fever. The intracellular bacteria, Wolbachia
is not correct, thus the answer will be option
cannot be transmitted to humans and acts
like a vaccine for the mosquito which ‘a’ and not ‘b’ or ‘c’.
carries dengue, Aedes aegypti, and stops the
virus multiplying in its body. The hope is they
will multiply, breed and become the majority Q6, September 22nd, 2016
of mosquitoes, thus reducing cases of the (Click Here): Regarding Wolbachia – It was a
disease.”
mistake we made while uploading the
http://www.bbc.com/news/world-asia-india- question. Option ‘c’ is supposed to be the right
37415781 answer, but the statement was supposed to
be, “A bacteria which acts like a vaccine for the
mosquito which carries dengue and stops
multiplying in its body”. For those who want to
*Clarifications* read about Wolbachia, it has already been
provided as a hyperlink in the explanation. You
can also click here.
Q4, Quiz, 21st September 2016
(Click Here): Regarding Montreal Protocol and
HFC – Technically, the MP is not yet dealing Notes
with HFC, though the parties in 2015 itself
agreed to amend the MP to address the issue
of HFCs which has zero Ozone Depleting
Potential. Thus, the answer should be option
‘c’, and not ‘b’. But for our purpose, before
pre-2017, the amendment to MP is likely to be
passed. Stay up-to-date, and we too will post
updated questions on the same issue when it
appears in the news once again.

Q4, September 21st, 2016


(Click Here): Regarding Montreal Protocol –
According to Britannica, “The CBDR….was
retroactively incorporated into the Vienna
Convention and Montreal Protocol on
substances that destroy the ozone layer.
Practically, it entails the deferral of developing
countries’ compliance with the objectives of
these environmental conventions.”
Additionally, according to the Ozone
Secretariat (Click Here) under the UNEP, “The
Montreal Protocol embodies key guiding
principles which are now recognized to be the
cornerstones of sustainable development

©www.InisightsIAS.com 31 ©www.InsightsonIndia.com
|

1. Consider the following list: 2. Which of the following statements with


reference to River Cauvery and its basin is/are
1. Daudhan dam not correct?
2. Makodia dam 1. The agricultural area dependent on it for
3. Barwa Sagar irrigation in Tamil Nadu exceeds that of
Karnataka’s.
The above list of dams/reservoirs are
associated with which of the following 2. The waters of the basin is shared between
projects? Karnataka, Tamil Nadu as well as Kerala
and Pondicherry.
(a) Ken-Betwa Link Project
3. The crop being grown predominantly in the
(b) Damanganga-Pinjal Link Project irrigated areas of Karnataka is ragi
whereas in Tamil Nadu it is paddy.
(c) Damodar Valley Corporation
Select the correct answer using the code given
(d) Narmada Valley Development Authority
below:

(a) 1 and 2 only


Solution: a.
(b) 2 only
India’s first inter-State river interlinking
(c) 2 and 3 only
project was given a go-ahead by the National
Board for Wildlife (NBWL) at a meeting chaired (d) 3 only
by Environment Minister Anil Madhav on
August 23, according to a report that was
made public on Tuesday. Solution: d.
This would be the first time that a river project This question has been framed not with a
will be located within a tiger reserve. prelims perspective but with a mains
The Rs. 10,000-crore Ken-Betwa project will perspective.
irrigate the drought-prone Bundelkhand
 Nearly 29 lakh acres of paddy crop in TN
region but in the process will also submerge
and an estimated 14 lakh acres of paddy
about 10% of the Panna Tiger Reserve in
and semi-dry crops in Karnataka are
Madhya Pradesh, feted as a model tiger-
currently dependent on Cauvery water.
conservation reserve.
 In its final order on February 5, 2007, the
The main feature of the project is a 230-km Tribunal awarded 419 TMC ft out of 740
long canal and a series of barrages and dams TMC ft to Tamil Nadu, 270 TMC ft to
connecting the Ken and Betwa rivers that will Karnataka, 30 TMC ft to Kerala and 7 TMC
irrigate 5 lakh hectares in Madhya Pradesh ft to Pondicherry. The remaining 14 TMC ft
and 14,000 hectares of Uttar Pradesh, in was reserved for environmental protection
Bundelkhand. The key projects are the and outflow to sea. (To understand why the
Makodia and Dhaudhan dams, the latter allocation has been skewed, read the
expected to be 77 m high and responsible for historical background to the dispute in the
submerging 5,803 hectares of tiger habitat in article, the link to which has been provided
the Panna tiger reserve. below.)
 Agriculture experts have pointed to the
Primary Reference: refusal of farmers in both states to move
http://www.thehindu.com/news/national/wi towards cultivating less water-intensive
ldlife-panel-clears-first-phase-of-kenbetwa- but profitable crops even in the drier years.
project/article9128892.ece “Almost all irrigated areas are growing
Secondary Reference: http://www.india- paddy. In unirrigated areas, ragi is the
wris.nrsc.gov.in/wrpinfo/index.php?title=Ken predominant crop. If the Kharif ragi could
-Betwa_Link be grown under irrigated conditions
instead of paddy, there would be saving in

©www.InisightsIAS.com 32 ©www.InsightsonIndia.com
|

water without any economic detriment to 4. With reference to ‘Ghatam’, a drum, consider
the farmers…,” a Fact Finding Committee the following statements:
reported on the cropping pattern in
Karnataka. 1. Unlike other Indian percussion
instruments, such as the tabla and
http://indianexpress.com/article/explained/ mridangam, it does not have a membrane
the-fight-over-the-cauvery-questions-of- over its mouth.
economy-and-culture-3043387/
2. The pitch and resonance of the instrument
is changed by the player by varying the
pressure of the pot against the stomach.
3. Consider the following list:
Which of the statements given above is/are
1. Rivery Cauvery correct?
2. Port of Kaveripoompattinam (a) 1 only
3. Kallanai Dam (b) 2 Only
4. Silappadikaram (c) Both 1 and 2
The above list is associated in one way or the (d) Neither 1 nor 2
other with which of the following dynasties of
South India?

(a) Vijayanagara Kingdom Solution: c.

(b) Pandyas “The ghatam is one of the most ancient


percussion instruments of South India. It is
(c) Chola Dynasty (Medival Cholas) a clay pot with narrow mouth. The pot is
(d) Sangam Cholas usually placed on the lap of the performer,
with the mouth facing the belly.”

“Ghatam produce a distinctive metallic


Solution: d. sound and are made in several sizes, each size
having a different pitch. As used in Karnatak
“The Tamil kingdoms that rose in the Cauvery
music, the ghatam is positioned with its
basin were early purveyors of irrigation
mouth pressed against the player’s stomach.
projects. The oldest dam on the river, Kallanai,
The player taps the surface of the ghatam with
upstream of the city of Trichy, is said to have
the fingers and the base of the palm and
been built by Karikala Chola, who ruled
changes the pitch and resonance of the
Cholamandalam (Early or Sangam Chola King)
instrument by varying the pressure of the pot
in the 2nd century. At the great Chola port city
against the stomach.”
of Kaveripoompattanam or Poompuhar (capital
of the Early Cholas and located at the end Primary reference:
point of River Cauvery), trading ships from https://www.britannica.com/art/ghatam ;
distant lands docked. The port, described in
great detail in Silappadikaram, has now been Improvisation: This article appeared in a
lost to the sea, and the river can be crossed on regional edition of the Hindu’s Friday review.
foot in the lean season.” 5. Kuchipudi, one of the classical styles of Indian
Primary Reference: dance, is a dance-drama tradition that
http://indianexpress.com/article/explained/ originated in which state?
the-fight-over-cauvery-jewel-among-maidens- (a) Orissa
the-life-force-of-civilisation/
(b) Andhra Pradesh
Secondary Reference: Page 88, Tamil Nadu
History Textbook, Class XI (c) Kerala

(d) Karnataka

©www.InisightsIAS.com 33 ©www.InsightsonIndia.com
|

Solution: b.

“Two decades later, Gurus Raja and Radha


Reddy look at their brainchild, the Natya
Tarangini Institute of Kuchipudi dance with a
sense of fulfilment as they get ready to host
Parampara, their annual festival which is also
into its 20th year. From a festival that focussed
and promoted Kuchipudi, a classical dance
specific to the land of the Telugus-Andhra
Pradesh, to a national and later an
international stage hosting the best in the
fields of classical dance and music.”

http://ccrtindia.gov.in/kuchipudi.php

This article appeared in a regional edition of


the Hindu’s Friday review.

6. The Rafale fighter deal between India and


France is expected to be signed soon. In this
context, which of the following statements
is/are correct?

1. The Indian Air Force’s fighter squadron


strength requirement will be met if the deal
is signed.

2. A key feature of the Rafale is its capability


to be equipped with the Beyond-Visual-
Range meteor air-to-air missile, with a Page 12, The Hindu, 23rd September 2016;
range in excess of 150 kilometers.

Select the correct answer using the code given Notes


below:

(a) 1 only

(b) 2 Only

(c) Both 1 and 2

(d) Neither 1 nor 2

Solution: b.

The Indian Air Force’s fighter squadron


strength requirement will still not be met even
if the deal is signed. The requirement is at least
42 squadrons, whereas it has 33 at present.
The Rafale deal will ensure 2 more squadrons
(each squadron consists of around 18
fighters). See image:

©www.InisightsIAS.com 34 ©www.InsightsonIndia.com
|

1. Consider the following statements: 2. ‘Panjnad’ is the name given to a few rivers of
the Punjab region. Which of the following are
1. The state with the longest coastline in the among these rivers?
country is Andhra Pradesh.
1. Indus
2. The coast of India falls within the tropical
as well as the subtropical region. 2. Satluj

Which of the statements given above is/are 3. Beas


incorrect? 4. Ravi
(a) 1 Only 5. Chenab
(b) 2 Only 6. Jhelum

(c) Both 1 and 2 Select the correct answer using the code given
below:
(d) Neither 1 nor 2
(a) 2, 3 and 4 only

(b) 1, 2, 3, 4 and 5
Solution: a.
(c) 2, 3, 4, 5 and 6
Gujarat has the longest coastline.
(d) None of the above

Solution: c.

“The Indus flows southward and receives


‘Panjnad’ a little above Mithankot. The
Panjnad is the name given to the five rivers of
Punjab, namely the Satluj, the Beas, the
Ravi, the Chenab and the Jhelum.”

Page 24, India Physical Environment, Class XI


NCERT

Improvisation:
http://indianexpress.com/article/explained/i
ndia-pakistan-relation-indus-water-treaty-
terrorism-3044967/

According to GKtoday, “entire coast of India


falls within tropics”, which is not technically
3. Which of the following rivers of the Indus River
correct. Boundaries of the tropics – The Tropic
of Cancer marks the northern edge, whereas system originate in China?
the Tropic of Capricorn marks the southern
edge. Kori creek and Sir creek (both, Gujarat)
area fall in the subtropical region, viz. beyond 1. Indus
the Tropic of Cancer.

Improvisation: 2. Satluj
http://pib.nic.in/newsite/PrintRelease.aspx?
relid=151074 3. Brahmaputra

4. Beas

©www.InisightsIAS.com 35 ©www.InsightsonIndia.com
|

Select the correct answer using the code given Solution: c.

below: Both statements are correct. Statement 2


though, is incomplete. Read the following
extract:
(a) 1, 2 and 3 only
“The Treaty gave India full rights over the
waters of the eastern rivers, while it had to let
(b) 1 and 2 only
the western rivers flow “unrestricted” to
Pakistan. India could use the waters of
(c) 3 and 4 only western rivers as well, but only in a “non-
consumptive” manner. It could use it for
domestic purposes, and even for irrigation and
(d) 1, 2, 3 and 4
hydropower production, but only in the
manner specified in the Treaty. With the
eastern rivers, India could do as it pleased.”

Solution: b. “We have never used our rights on the western


rivers. Under the Treaty, we can make use of
“The Treaty, which came after a decade of the waters of the western rivers for irrigation,
World Bank-brokered negotiations, classified storage, and even for producing electricity, in
the six rivers of the Indus system into ‘eastern’ the manner specified. If we just do what we are
and ‘western’ rivers. Sutlej, Beas and Ravi entitled to under the Treaty, it would be
were eastern; Jhelum, Chenab and Indus itself enough to send jitters through Pakistan. It
were western. The categorisation was relative would be a strong signal without doing
— the western rivers flow almost parallel to the anything drastic,” Sinha said.
west of the eastern ones. Indus, the largest
river, originates in China, so does the Sutlej. http://indianexpress.com/article/explained/i
The other four rise in India; all enter Pakistan ndia-pakistan-relation-indus-water-treaty-
from India.” terrorism-3044967/

http://indianexpress.com/article/explained/i
ndia-pakistan-relation-indus-water-treaty-
terrorism-3044967/ 5. “Friendship 2016” is a military exercise which
will see the participation of which of the
following nations?

4. Consider the following statements with (a) Russia and Pakistan


reference to some provisions in the 1960 Indus
Waters Treaty: (b) India and Russia

1. With the Eastern Indus Rivers, India is (c) China and Pakistan
entitled to do as it pleases – dam the (d) Russia, China and India
waters, produce electricity, among other
possible activities.

2. India can make use also of the waters of Solution: a.


the Western Indus Rivers for irrigation,
“Shortly after Russian news agency Tass
storage, and even for producing electricity.
reported that the first-ever joint military
Which of the statements given above is/are exercise between Russia and Pakistan
correct? (dubbed, “Friendship 2016”) will open at Rattu
in Gilgit-Baltistan, the Russian Embassy in
(a) 1 only Delhi issued a strong denial calling the reports
(b) 2 Only “erroneous” and “mischievous”….”

(c) Both 1 and 2 “The eighth edition of India-Russia Joint


Military Exercise ‘INDRA-2016’ has begun in
(d) Neither 1 nor 2 the Ussiriysk District in Vladivostok, Russia.

©www.InisightsIAS.com 36 ©www.InsightsonIndia.com
|

Main focus of this edition of the joint exercise


is on ‘Counter-Terrorism Operations in semi
mountainous and jungle terrain under United Notes
Nations Mandate’.”

http://indianexpress.com/article/world/worl
d-news/no-joint-military-exercise-with-
pakistan-in-pok-russia-clarifies-3047234/

http://www.thehindu.com/news/national/ru
ssiapakistan-joint-war-games-begin-
today/article9141394.ece

6. ‘Aleppo’, seen often in the news, is a city in

(a) Syria

(b) Turkey

(c) Iraq

(d) Jordan

Solution: a.

http://www.thehindu.com/news/internation
al/fierce-air-strikes-on-aleppo-after-syrian-
army-declares-offensive/article9140699.ece

©www.InisightsIAS.com 37 ©www.InsightsonIndia.com
|

1. The norms recommended by the Indian  children’s education,


Labour Conference, held in 1957, are taken  medical requirements,
into account while fixing the minimum wages  minimum recreation including
to this day. These recommendations include festivals/ceremonies,
which of the following?  provision for old age,
1. Minimum food requirements on the basis  provision for marriage etc.
of calories required per average Indian should further constitute 25% of the minimum
adult. wage and be used as a guide in fixation of
2. Minimum recreation including minimum wage.”
festivals/ceremonies. September 24th, PIB Features: Understanding
3. Cloth requirements per annum per family. Minimum Wages and Bonus.

4. Children’s education.

Select the correct answer using the code given 2. There are 45 scheduled employments in the
below: sphere of the Central Government. The
minimum wages for these scheduled
(a) 1, 2 and 3 only employments are fixed by

(b) 1 and 3 only (a) The respective ministries

(c) 2 and 4 only (b) The Ministry of Labour and Employment

(d) 1, 2, 3 and 4 (c) The Ministry of Finance

(d) None of the above

Solution: b.

Keep in mind the context – the ILC norms were Solution: d.


recommended in 1957 – so the very basic
needs (clothing, a roof over the head, food, fuel Fixing minimum wages and revising them are
to cook food) were sought to be guaranteed decisions taken by the Central Government,
back then, whereas by 1991/1992, what and not any one ministry.
constituted the basic requirements had Economic Times, August 31st: “Finance
expanded. This way, you can easily answer Minister Arun Jaitley, who chairs the inter-
this question. ministerial committee on labour, said the
“The norms recommended by the Indian decisions are based on the proposals of the
Labour Conference, in 1957, fox fixing the panel set up last year to look into the 12-point
minimum wages are: charter of demands of the trade unions.”

Arthapedia: “Minimum wage corresponds to


 consumption units for one wage earner;
the minimum cash payment to be paid to an
 minimum food requirements of 2700
individual working in certain notified jobs
calories per average Indian adult;
(mostly in the informal / unorganised sector),
 clothing requirements of 72 yards per
at a rate fixed by the central or state
annum per family;
Governments.”
 rent corresponding to the minimum area
provided for under Government’s “The Minimum Wages Act, 1948 envisages
Industrial Housing Scheme; and appointment of an Advisory Board, by the
 fuel, lighting and other miscellaneous appropriate Government, for the purpose of
items of expenditure to constitute 20% of advising the appropriate Government in the
the total minimum wage.” matter of fixing and revising minimum rates of
wages.”
“In 1991 (1992 according to Arthapedia), the
Hon’ble Supreme Court delivered a historic September 24th, PIB Features: Understanding
judgement and directed that Minimum Wages and Bonus.

©www.InisightsIAS.com 38 ©www.InsightsonIndia.com
|

3. Whereas revision of minimum wages for the Solution: a.


scheduled employments is done from time to
time in accordance with the provisions of the “In the hills and forests of Jharkhand, Sarhul
Minimum Wages Act, 1948, bonus payment – marks the beginning of the New Year. It is
extra payment given for doing one’s job well – celebrated by the Oraon, the Munda, the Ho,
is granted in accordance with the provisions of the Santhal, and by us, the Birjia. Falling on
the third day of the moon in the month of
(a) Minimum Wages Act, 1948 Chaitra, Sarhul marks the advent of spring,
and the word means the worship of the sal
(b) Payment of Bonus Act, 1965 tree.”
(c) The Supreme Court directive in ‘Reptakos
 Sarhul honours the return of Mother
& Co. versus its workers’ in 1992.
Nature’s daughter Bindi to earth.
(d) It is an executive decision of the Central  Sarhul is celebrated during spring season
Government. and the Saal trees get new flowers on their
branches.
 It is also a worship of the village deity who
Solution: b. is considered to be the protector of the
tribes. The deities are worshipped with Sal
“Bonus payment is an extra payment given for flowers.
doing one’s job well also known as
performance-related pay or pay for Primary reference:
performance. The practice of paying bonus in http://www.thehindu.com/features/magazin
India appears to have originated during First e/amrita-besra-on-the-traditions-associated-
World War when certain textile mills granted with-the-sarhul-festival-of-
10% of wages as war bonus to their workers in jharkhand/article9140459.ece
1917.” Secondary reference:
“The objective of the Payment of Bonus Act, http://www.jharkhand.gov.in/festivals
1965 is to provide for the payment of bonus to
the persons employed in certain
establishments on the basis of profits or on the 5. Consider the following statements with
basis of production or productivity and for reference to the Income Declaration Scheme,
matter connected therewith.” 2016:

September 24th, PIB Features: Understanding 1. The scheme is applicable to undisclosed


Minimum Wages and Bonus. income – in the form of assets or otherwise
– pertaining to the financial year 2015-16
or earlier.
4. ‘Sarhul’ is a festival which is celebrated by 2. Those with undisclosed income can
certain tribes declare it by paying a penal tax rate of 45%
(a) In the Jharkhand region and marks the on such income.
beginning of their New Year. 3. The information in respect of a valid
(b) Belonging to the State of Meghalaya and is declaration will not be enquired into by the
a thanksgiving for nature’s bounty. Income-tax Department or any law
enforcement agency.
(c) In the Niligiris who worship the Sarhul
Devta – the God of power, youth and Which of the statements given above is/are
youthfulness. correct?

(d) Of Madhya Pradesh marking the beginning (a) 1 and 2 only


of ploughing and praying for a good (b) 2 and 3 only
agricultural season.
(c) 1 and 3 only

(d) 1, 2 and 3

©www.InisightsIAS.com 39 ©www.InsightsonIndia.com
|

Solution: c. such as lower customs duty rates can be


granted to the MFN nation.
“ In a series of tweets, Revenue Secretary
Hasmukh Adhia said that expectations are (c) Countries cannot normally discriminate
raised that looking to the enthusiasm of people between their trading partners.
for making IDS declaration, the date for
income disclosure scheme (IDS) would be (d) ‘X’ nation with which ‘Y’ nation accounts
extended. “We want to make it very clear that for the largest share in value of the former’s
the last date for Income Declaration Scheme foreign trade shall be granted special
(IDS) will not be extended,” Mr. Adhia favours, in accordance with MFN norms,
tweeted.” by the latter.
Regarding the non-disclosure of information to
the IT Dept or any law enforcement agency, the
declaration must be a valid declaration. Solution: c.
Additionally, an associated point, regarding
“Under the WTO agreements, countries cannot
disclosure (and not investigation) – “bringing
normally discriminate between their trading
in section 138 of the IT Act is to dispel the
partners. Grant someone a special favour
perception that the details disclosed under the
(such as a lower customs duty rate for one of
scheme are confidential under any
their products) and you have to do the same
circumstances — they can be disclosed if it is
for all other WTO members.”
deemed to be in public interest. The key to
this, according to tax experts, is the objectivity “This principle is known as most-favoured-
of the Chief Commissioner.” nation (MFN) treatment (see box). It is so
important that it is the first article of the
“Such income as declared, would be taxed at
General Agreement on Tariffs and Trade
the rate of 30% plus a ‘Krishi Kalyan Cess’ of
(GATT), which governs trade in goods. MFN is
25% on the taxes payable (equivalent to 7.5%
also a priority in the General Agreement on
of the undisclosed income) and a penalty at the
Trade in Services (GATS) (Article 2) and the
rate of 25% of the taxes payable (equivalent to
Agreement on Trade-Related Aspects of
7.5% of the undisclosed income and not 45%),
Intellectual Property Rights (TRIPS) (Article 4),
thereby totalling to 45% of the income declared
although in each agreement the principle is
under the scheme.”
handled slightly differently. Together, those
Primary reference: three agreements cover all three main areas of
http://pib.nic.in/newsite/PrintRelease.aspx? trade handled by the WTO.”
relid=145360
“Some exceptions are allowed. For example,
Secondary reference: countries can set up a free trade agreement
http://pib.nic.in/newsite/PrintRelease.aspx? that applies only to goods traded within the
relid=149530 group — discriminating against goods from
outside. Or they can give developing countries
special access to their markets.”
6. The principle known as ‘Most-Favoured- Primary reference:
Nation’ or MFN, under the General Agreement https://www.wto.org/english/thewto_e/what
on Tariffs and Trade, essentially means that is_e/tif_e/fact2_e.htm

(a) Only one country can be designated as Source for question:


MFN by another nation as per its http://www.thehindu.com/business/Econom
discretion, under which special favours y/mfn-status-to-pakistan/article9144854.ece
such as lower customs duty rates can be
granted to the MFN nation.

(b) Any number of nations can be granted the


MFN status by another nation as per its
discretion, under which special favours

©www.InisightsIAS.com 40 ©www.InsightsonIndia.com
|

7. ‘PROVe’ recently seen in the news, is the name polymetallic nodules and hydrothermal
of a Remotely Operated Vehicle (ROV). It sulphides, which occur at water depths
serves/served what purpose? ranging between 1,000 and 6,000 metres, said
a communication from the institute.”
(a) It played a key role in the discovery of large
amounts of potentially producible gas http://www.thehindu.com/todays-paper/tp-
hydrates in the Bay of Bengal. features/tp-educationplus/indian-rov-
monitors-the-health-of-coral-
(b) India’s first seabed ROV in the arctic region reefs/article9147486.ece
for continuous monitoring of
oceanographic parameters, to understand
the Arctic climate and it's possible link to
the Indian monsoon. 8. ‘iKnife’, recently seen in the news, is

(c) It is deployed by the International Seabed (a) The Council of Scientific and Industrial
Authority to explore deposits of Research’s National Facility for validation
Polymetallic Nodules. of solar cell efficiency which is considered
one of the world’s most accurate validation
(d) An indigenously-developed Remotely processes and systems established.
Operated Vehicle, to map the coral reefs in
Andaman & Nicobar Islands. (b) A surgical knife, which tests tissue as it
contacts it during an operation, and
immediately gives information as to
whether that tissue contains cancer cells.
Solution: d.
(c) The Genome project initiated by the
“In 2015, ‘PROVe’ was in the news as a ‘Polar Cellular and Molecular Biology Institute to
Remotely Operated Vehicle’ which “was sequence the ‘letters’ making up the
operationalised for research in North human DNA.
Antarctica in Priyadarshini Lake on the
Schirmacher Oasis, which is the source of (d) None of the above.
water for Maitri, India’s second base in the icy
continent.” Solution: b.

This PIB Release from 2016 refers to PROVe in What is the ‘iKnife’?
the following manner: “Currently, there is no “Cancer detection techniques, during surgery,
mechanism other than Scuba diving to have become good enough to pinpoint a
examine the corals and assess the extent of tumour but neither images nor the human eye
damage or rejuvenation that might be taking can quickly tell apart healthy and unhealthy
place since the great damages happened tissues. That’s where the iKnife comes handy.
earlier. National Institute of Ocean Based on electrosurgery — a technology
Technology(NIOT), for the first time, used the around since 1920s that employs electric
indigenously-developed Remotely Operated current to heat up the tissue that needs
Vehicle (PROVe), to map the coral reefs in excision — the iKnife can detect precisely
Andaman & Nicobar Islands (North Bay and which tissue needs removing, and which
Chidiyatapu) and that the ROV can be used for should stay.”
this purpose efficiently.” So it most likely is the
same PROVe from 2015 now being used in the
Andamans.
9. “Exomoons” refer to
Additionally, “NIOT had earlier developed a
(a) Moons of Exoplanets.
deep water work class Remotely Operated
Vehicle (ROV) ROSUB 6000 which was (b) Moons of dwarf planets.
suitable for exploration in deep waters. It was
successfully operated at a maximum depth at (c) Moons discovered in galaxies other than
5,289 metres in the Central Indian Ocean the Milky Way.
Basin. It also contributed to the exploration of
(d) Another name given to Exoplanets that
deep ocean minerals such as gas hydrates,
orbit Red Dwarfs.

©www.InisightsIAS.com 41 ©www.InsightsonIndia.com
|

Solution: a. 11. ‘Tianyan’, seen in the news recently, refers to


China’s
“The recent discovery of Proxima b, an
exoplanet in our neighbouring star system, (a) Military-backed space programme.
Proxima Centauri in the Alpha Centauri group,
has fuelled popular interest in studying new (b) Renewed push through investments in
worlds. However, theoretical and observational innovation and technological research to
study of exoplanets – planets orbiting stars become the world’s ‘ultimate tourist
other than the sun – is not new. Among the destination’.
exoplanets discovered many cannot (c) First experimental Space Station which is
themselves support life. But it is expected that set to fall back to Earth in 2017.
these could be surrounded by huge natural
satellites having water and which are perhaps (d) Radio telescope which is the world’s
even habitable. This is a reason to look for largest.
such friendly “exomoons” as well as
exoplanets.”
Solution: d.
http://www.thehindu.com/todays-paper/tp-
features/tp-sci-tech-and-agri/a-new- “The world’s largest radio telescope began
technique-to-look-for- searching for signals from stars and galaxies
exomoons/article9146210.ece and, perhaps, extraterrestrial life on Sunday in
a project demonstrating China’s rising
ambitions in space and its pursuit of
10. Section 52(1) of the Copyright Act, 1957 deals international scientific prestige.”
with
“The official Xinhua News Agency said
(a) The duration of copyright allotted to hundreds of astronomers and enthusiasts
different works – literary, dramatic, watched the launch of the Aperture Spherical
musical and artistic works. Telescope, or FAST, in the county of Pingtang.”

(b) The remedies available to owners of “Installation of the 4,450-panel structure,


copyright for instances of its infringement. nicknamed Tianyan, or the Eye of Heaven,
started in 2011 and was completed in July.”
(c) Exceptions to what constitutes ‘copyright
infringement’. http://www.thehindu.com/news/internation
al/worlds-largest-radio-telescope-begins-
(d) The rights in India of foreign creators and operations/article9147058.ece
owners of copyrights.

Solution: c. NOTES
Section 52(1) in the Copyright Act, 1957

Relevant for our purpose: Section 52(1)(i) – The


following acts shall not constitute an
infringement of copyright, namely:

 by a teacher or a pupil in the course of


instruction; or
 as part of the questions to be answered in
an examination; or
 in answers, to such questions;

On the right to photocopy

©www.InisightsIAS.com 42 ©www.InsightsonIndia.com
|

1. Consider the following list: 2. Why did Mahatma Gandhi change his attire to

1. Ksheer Scanner a simple dhoti and shawl on September 22nd,


1921?
2. BGR-34

3. India’s first parallel computer (a) Khadi was to him a sign of poverty.

4. Drishti system (b) He believed it was a statement of boycott of


The above list of foreign cloth, and he estimated that his
systems/devices/formulations are associated
stature among the masses would convince
with which of the following entities?
them to adopt a similar attire.
(a) Council of Scientific and Industrial
Research (c) Foreign cloth was not affordable but had
(b) Department (now Ministry) of Electronics gained currency only due to the ‘elite’
and Information Technology status associated with it; he sought to turn
(c) Indian Institute of Science this perception on its head.
(d) Centre for Development of Advanced
(d) Both b and c.
Computing

Solution: a. Solution: a.

“From producing the indelible ink that has He made no such ‘estimation’ of his stature.
been worn by millions of fingernails since the But he did believe that it was an act of boycott
first general election in 1952 to the country’s of foreign cloth. “However, Gandhiji did not
first parallel supercomputer in the late 80s, want everyone to follow his simplistic dress
the Council of Scientific & Industrial Research style. He wrote in Navajivan: “I do not want
(CSIR) has come a long way in seven decades either my co-workers or readers to adopt the
of its existence.” loincloth. But I do wish that they should
thoroughly realise the meaning of the boycott
 CSIR has launched an anti-diabetic herbal of foreign cloth and put forth their best effort
formulation, namely, BGR-34 from a to get it boycotted, and to get khadi
combination of natural extracts derived manufactured. I do wish that they may
from six plant species mentioned in understand that swadeshi means everything.”
ancient Ayurveda texts.
 As high levels of milk contamination came “At this time, he did not want to use this dress
to notice, CSIR developed a system, all his life and only wanted to ‘experiment for
Ksheer-Scanner which detects a month or two’. But soon he saw this as his
adulteration of milk by the use of urea, duty to the poor, and he never wore any other
salt, detergent, liquid soap, boric acid. dress. He consciously rejected the well-known
 CSIR has installed Drishti systems at clothes of the Indian ascetic and adopted the
Delhi’s busy IGI Airport for providing dress of the poorest Indian. Khadi, white and
information to pilots on visibility for safe coarse, was to him a sign of purity, of simplicity,
landing and take-off operations. and of poverty. Wearing it became also a
symbol of nationalism, a rejection of Western
16th September, PIB Features: From Indelible mill- made cloth.”
Ink to Parallel Supercomputer: CSIR partners
in National Development Additionally, “Those who had been deprived by
caste norms for centuries were attracted to
Western dress styles. Therefore, unlike
Mahatma Gandhi, other nationalists such as
Babasaheb Ambedkar never gave up the
Western-style suit. Many Dalits began in the

©www.InisightsIAS.com 43 ©www.InsightsonIndia.com
|

early 1910s to wear three- piece suits, and  “Some of the earliest works in Tamil,
shoes and socks on all public occasions, as a known as Sangam literature, were
political statement of self-respect.” composed around 2300 years ago. These
texts were called Sangam because they
Primary Reference: Page 174-176, India and
were supposed to have been composed and
Contemporary World – I, Class 9 NCERT
compiled in assemblies (known as
Improvisation: September 16th, PIB Features: sangams) of poets that were held in the city
What made Gandhiji wear only Loincloth or of Madurai.”
Dhoti.  A Hundred Years ago, i.e. on September
16, 1916, ‘M.S. Subbulakshmi, the
‘Nightingale of India’, was born at Madurai
in Tamil Nadu.
3. Consider the following list of statements:
Page 89 & 127, Our Pasts – I, Class 6, NCERT
1. The ‘Nightingale of India’ was born here.
September 14th, PIB Features: The inimitable
2. Gandhiji’s decision to adopt, and his first
M.S.Subbulakshmi
public appearance in, the attire of a dhoti
and shawl was here.

3. Some of the earliest works in Tamil were 4. Engineers’ Day in India is observed each year
supposed to have been composed and on September 15th in the honour of
compiled in assemblies that were held in
this city. (a) Kalpana Chawla

4. Kannagi, in the famous Tamil epic (b) Mokshagundam Visvesvaraya


Silappadikaram, destroys this entire city.
(c) Homi Jehangir Bhabha
The above statements refer to which city from
(d) Chandrasekhara Venkata Raman
among the following?

(a) Kanchipuram
Solution: b.
(b) Kaveripattinam
“September 15 is observed as the Engineers’
(c) Madurai
Day in the honour of Dr. Mokshagundam
(d) Karur Visvesvaraya (1861-1962), India’s pioneering
engineer and administrator, who was born on
that date.”
Solution: c. “M. Visvesvaraya stands out as the pioneering
Indian engineer. He began as a civil engineer
Statement 3 or 4 should have made this
in Bombay Presidency in 1884. But it was his
question easy.
engineering feat at Sukur in Sindh Province
 “A famous Tamil epic, the (then part of Bombay Presidency) in 1893-
Silappadikaram, was composed by a 1895 that brought him fame. He devised an
poet named Ilango, around 1800 years ago. ingenious method to filter the muddy and
It is the story of a merchant named discoloured water of Indus by digging a well on
Kovalan, who lived in Puhar and fell in love riverbed and creating a tunnel below the river
with a courtesan named Madhavi, to bring drinking water for the city to the
neglecting his wife Kannagi. Later, he and pumping station. Later on, he specialized in
Kannagi left Puhar and went to Madurai, irrigation, water supply and drainage works.
where he was wrongly accused of theft by He was involved in Pune suburban water
the court jeweller of the Pandya king. The supply project. He devised sluice gates at Lake
king sentenced Kovalan to death. Kannagi, Fife in Khadakvasala, Pune for which he took
who still loved him, was full of grief and patent. This technology stopped the loss of
anger at this injustice, and destroyed the excess water. He also lent consultancy services
entire city of Madurai.” to Nizam of Hyderabad after the great Musi

©www.InisightsIAS.com 44 ©www.InsightsonIndia.com
|

floods of 1908. The two projects he advised the least 55 per cent of global greenhouse gas
Nizam government was related to sewerage emissions. Both conditions need to be met.”
and reservoir of river Musi, a tributary of
Krishna in 1909. The role played by Dr. Indian Express: April 25th, 2016
Visvesvaraya in setting up of Hindustan http://indianexpress.com/article/explained/
Aeronautical Limited in Bangalore in 1940 is paris-climate-deal-a-shift-in-position-by-
less discussed.” india/
 Visvesvaraya served as the chief engineer
during the construction of the KRS:
“Krishna Raja Sagara, also popularly 6. The ‘FARC’ rebels, who were known for
known as KRS, is the name of both a lake infamous kidnappings, guerrilla attacks and
and the dam that creates it. The dam is military onslaughts, recently signed a historic
across Cauvery River, in Mandya in treaty marking an end to the armed conflict
Karnataka state, India.” .. Mandya is often with the government of which of the following
an important region for protests by farmers countries?
whenever the Cauvery River Water-sharing
(a) Central African Republic
issue gathers steam.
(b) Ecuador
September 14th, PIB Features: Engineers are
Nation Builders. (c) Columbia

(d) Peru
5. With reference to the Paris Agreement on
Climate Change, which 174 countries and the
European Union signed recently, which of the Solution: c.
following statements is/are incorrect? ‘FARC’ when expanded and translated to
1. In India, approval of the Parliament will be English, is “The Revolutionary Armed Forces of
required for the government to ratify it. Colombia”.

2. It requires the ratification by at least 55 “All bad things come to an end. And so, after
countries which together account for at 52 years of bloody clashes, guerilla attacks,
least 55 per cent of global greenhouse gas military onslaughts and infamous
emissions, for its operationalisation. kidnappings, the war between Colombia’s
FARC — aka The Revolutionary Armed Forces
Select the correct answer using the code given of Colombia — and the government has ended.
below: The two sides signed a historic treaty, FARC
agreeing to end its armed conflict and enter the
(a) 1 only
legal political process.”
(b) 2 only
http://indianexpress.com/article/opinion/ed
(c) Both 1 and 2 itorials/farc-drugs-colombia-treaty-peace-
end-armed-conflict-3051595/
(d) Neither 1 nor 2

7. Further evidence has recently been obtained to


Solution: a. show that the icy moon ‘Europa’ throws jets of
water out into space. Which planet does this
The same question was posted here a few
moon orbit?
months ago. “In India, approval of Parliament
will not be required for the government to ratify (a) Neptune
the Paris Agreement. A Cabinet decision to this
effect would be enough…. For its (b) Uranus
operationalisation, the Paris Agreement
(c) Saturn
requires the signing and ratification by at least
55 countries which together account for at (d) Jupiter

©www.InisightsIAS.com 45 ©www.InsightsonIndia.com
|

Solution: d. 9. The military exercise ‘Yudh Abhyas-2016’ is a


joint military training exercise, to hone the
“Scientists first reported the behaviour in 2013 skills in counter-terrorism operations,
using the Hubble telescope, but have now between
made a follow-up sighting. It is significant
because Europa, with its huge subsurface (a) India and Russia.
ocean of liquid water, is one of the most likely
places to find microbial life beyond Earth. (b) India and United States of America.
Flying through the jets with an instrumented (c) India, USA and Japan.
spacecraft would be an effective way to test the
possibility. One could even attempt to capture (d) The three wings of the Indian Armed
a sample of ejected material and bring it back Forces.
to Earth for more detailed biological analysis.”

“Water jets have already been seen up close at


Solution: b.
Enceladus, an icy moon of Saturn. These
emanate from a series of fissures at its south “Over the duration of past two weeks, Armies
pole.” of India and USA have participated in Exercise
Yudh Abhyas at Chaubattia. The exercise
http://www.bbc.com/news/science-
aimed at building positive military relations
environment-37473617
between the two countries and allowed the two
armies to understand each other’s tactical and
operational methodologies.”
8. Which of the following reasons is most
appropriate explanation for water shortage http://pib.nic.in/newsite/PrintRelease.aspx?
being experienced in India today? relid=151121

(a) Over-use, injudicious exploitation, and


wastage.
Notes
(b) Population pressures and lifestyle-induced
increases in demand.

(c) Unpredictable monsoons as a result of


Climate Change.

(d) Human-induced Water Pollution.

Solution: b.

Not exactly a preliminary-type question, but


this Indian Express article is certainly helpful.

“A common perception is that water shortage


is mainly a result of over-use, injudicious
exploitation, and wastage. While these have
certainly aggravated the situation, water
availability per capita has reduced sharply
over the last century due to population
pressures and lifestyle-induced increases in
demand. According to one estimate, water
demand has grown 6 times since 1930, while
population has grown by slightly more than 3
times. Water availability would, therefore,
have gone down even if it had been used in the
most efficient manner possible.”

©www.InisightsIAS.com 46 ©www.InsightsonIndia.com
|

1. Which one of the following issues the “Global (c) Thiruvananthapuram, Kerala
Competitiveness Report” periodically?
(d) Chandipur, Odisha
(a) The World Bank

(b) The European Bank for Reconstruction


and Development Solution: b.

(c) Organisation for Economic Cooperation “The 371 kg SCATSAT-1, a satellite for
and Development weather-related studies, was placed in the
polar sun synchronous orbit at an altitude of
(d) The World Economic Forum 730 km some 17 minutes after the rocket took
off from Satish Dhawan Space Centre at
Sriharikota at 9.12 a.m.”
Solution: d. Primary reference:
“India has risen rapidly among all countries in http://www.isro.gov.in/about-isro/launch-
the global competitive stakes by climbing 16 complex
notches to the 39th position during the past Source: http://www.thehindu.com/sci-
year in the WEF’s Global Competitiveness tech/science/isros-pslvc35-places-scatsat1-
Index. According to the World Economic into-orbit-on-its-way-to-place-seven-
Forum’s (WEF) latest Global Competitiveness others/article9149124.ece
Report for 2016-17 this marks the biggest
scale of improvement in competitiveness
among all countries and is the second year in
a row India has gone up 16 ranks in the WEF 3. What were some of the unique aspects of
index.” ISRO’s PSLV C-35 rocket launch?

“The Global Competitiveness Report 2016- 1. It was the longest PSLV launch by ISRO.
2017 assesses the competitiveness landscape 2. This was the first mission of PSLV in which
of 138 economies, providing insight into the it had launched payloads into two different
drivers of their productivity and prosperity. orbits.
This year’s edition highlights that declining
openness is threatening growth and 3. This is by far the most number of satellites
prosperity. It also highlights that monetary to be launched in one PSLV rocket launch.
stimulus measures such as quantitative
Select the correct answer using the code given
easing are not enough to sustain growth and
below:
must be accompanied by competitiveness
reforms. Final key finding points to the fact (a) 1 and 2 only
that updated business practices and
investment in innovation are now as important (b) 2 and 3 only
as infrastructure, skills and efficient markets.”
(c) 1 and 3 only
http://www.thehindu.com/business/Econom
(d) 1, 2 and 3
y/world-economic-forum-india-now-ranks-
39-among-most-competitive-
economies/article9155344.ece
Solution: a.

The PSLV C-34 recently launched 20 satellites


2. The ‘Satish Dhawan Space Centre’, also known in a single flight. Only statements 1 and 2 are
as the ‘Spaceport of India’, carries out all correct.
operations related to ISRO’s launches. It is
located at “In a first, Indian Space Research Organisation
(ISRO)’s PSLV C-35 rocket launched a total of
(a) Abdul Kalam Island, Odisha eight satellites (including two satellites from
two educational institutions), into two
(b) Sriharikota, Andhra Pradesh
different orbits. Announcing the successful

©www.InisightsIAS.com 47 ©www.InsightsonIndia.com
|

launch of all the satellites from the Mission http://www.thehindu.com/opinion/editorial/


Control Centre, ISRO chairman A.S. Kiran editorial-on-indus-water-treaty-troubled-
Kumar said the Monday launch marked a waters/article9154697.ece
“landmark day” in the history of ISRO. The
rocket was re-ignited twice during its flight to
place the set of satellites in different orbits. 5. Of all of pollutants, fine particulate matter has
Due to the re-ignition, the Monday’s launch is the greatest impact on health. The World
by far the longest PSLV launch by ISRO. ISRO Health Organisation’s threshold values for
said though it had launched several PSLV PM2.5 particles measured in micrograms per
rockets in the past, this launch is “the first cubic metre, for a 24-hour mean is:
mission of PSLV in which it had launched its
payloads into two different orbits,” ISRO said.” (a) 10

Additionally, “Since 2009, the Indian Space (b) 25


Research Organisation (ISRO) has encouraged
(c) 50
and mentored students to acquire experience
in satellite development and has allowed such (d) None of the above.
satellites to ride piggy back for free, along with
commercial payloads.”

http://www.thehindu.com/sci- Solution: d.
tech/science/isros-pslvc35-places-scatsat1-
The purpose of the question was to make you
into-orbit-on-its-way-to-place-seven-
aware that there is no ”threshold” value.
others/article9149124.ece
“Small particulate pollution have health
impacts even at very low concentrations –
4. Negotiations on sharing of the Indus Waters indeed no threshold has been identified below
between India and Pakistan culminated in the which no damage to health is observed.
signing of the Indus Waters Treaty in 1960. Therefore, the WHO 2005 guideline limits
Under the watch of which body/country were aimed to achieve the lowest concentrations of
these negotiations held? PM possible. Guideline values:

(a) United Nations Security Council  5: 10 μg/m3 annual mean; 25 μg/m3 24-
hour mean;
(b) The Chair of the Non-Aligned Movement at  PM10: 20 μg/m3 annual mean; 50 μg/m3
the time. 24-hour mean;
(c) International Bank for Reconstruction and “In instances where accurate PM2.5 (that is,
Development 2.5 micrometers or less) measurements were
(d) None of the above. unavailable, the researchers derived their
averages based on PM10, which are larger dust
particle-concentrations. It notes that more
than 80 per cent of people living in urban areas
Solution: c. that monitor air pollution are exposed to air
The same question was posted here last quality levels that exceed the World Health
month. Organization (WHO) limits. The study gave the
WHO air quality guidelines for PM2.5 as 10
“More important, the 1960 treaty for the Indus micrograms per cubic metre annual average,
and five tributaries flowing from India to and 25 micrograms per cubic metre 24-hour
Pakistan was brokered by the World Bank average.”
(then, the IBRD), and has held through wars
and conflicts along the Line of Control. Primary Reference:
Revoking it would threaten regional stability http://www.who.int/mediacentre/factsheets
and India’s credibility globally.” /fs313/en/

http://wrmin.nic.in/forms/list.aspx?lid=346 http://www.thehindu.com/sci-tech/energy-
and-environment/india-accounts-for-75-of-

©www.InisightsIAS.com 48 ©www.InsightsonIndia.com
|

air-pollution-casualties- 7. With reference to the Monetary Policy


who/article9154552.ece Committee (MPC), which of the following
statements is/are incorrect?

1. Altogether, it will have seven members.


6. Consider the following list:
2. The Reserve Bank of India (RBI) Governor
1. Kokborok will not vote in the first instance, viz. he
2. Bodo possesses the deciding vote in case of a tie.

3. Maithili 3. All central government nominees of the


MPC will hold office for a period of four
4. Vajjika years and will not be eligible for re-
appointment.
Which languages from the above list are listed
under Schedule 8 of the Indian Constitution? Select the correct answer using the code given
below:
(a) 1 and 2
(a) 1 only
(b) 2 and 3
(b) 1 and 2
(c) 3 only
(c) 3 only
(d) 1 and 4
(d) 2 and 3

Solution: b.
Solution: b.
“NBT will organise a Bodo workshop soon, and
this too will be aimed at identifying what books Question being repeated.
can be published in the language used by the
Bodo community in Assam. The Devanagari  Altogether, the MPC will have six
script has been generally used for writing the members, – the RBI Governor
language – one of the 22 scheduled languages (Chairperson), the RBI Deputy Governor in
of India – since 1963. It has also brought out charge of monetary policy, one official
books — on society, Mahatma Gandhi, nominated by the RBI Board and the
children’s themes and so on — in Maithili, a remaining three members would represent
scheduled language of India spoken in north the Government of India.
Bihar, and Magahi and Bhojpuri, both dialects  These Government of India nominees are
used in Bihar.” appointed by the Central Government
based on the recommendations of a search
“The Eighth Schedule to the Indian cum selection committee consisting of the
Constitution contains a list of 22 scheduled cabinet secretary (Chairperson), the RBI
languages viz. Assamese, Bengali, Bodo, Governor, the secretary of the Department
Dogri, Gujarati, Hindi, Kannada, Kashmiri, of Economic Affairs, Ministry of Finance,
Konkani, Maithili, Malayalam, Manipuri, and three experts in the field of economics
Marathi, Nepali, Odia, Punjabi, Sanskrit, or banking as nominated by the central
Santali, Sindhi, Tamil, Telugu, Urdu. The list government.
had originally 14 languages only.”  The three central government nominees of
Primary reference: the MPC appointed by the search cum
http://www.thehindu.com/news/national/ot selection committee will hold office for a
her-states/in-kokborok-prologue-to-a-new- period of four years and will not be eligible
imprint/article9154871.ece for re-appointment.
 The RBI Governor will vote in the first
Secondary reference: instance. In case of a tie, the RBI governor
http://www.gktoday.in/blog/official- will have the second or casting vote. The
languages-constitutional- decision of the Committee would be
provisions/#Scheduled_languages binding on the RBI.

©www.InisightsIAS.com 49 ©www.InsightsonIndia.com
|

Primary Reference: Monetary Policy http://www.thehindu.com/opinion/editorial/


Committee editorial-on-organisation-for-economic-
cooperation-and-development-falling-behind-
Improvisation: schedule/article9146983.ece
http://www.thehindu.com/business/Econom
y/mpc-will-meet-before-october-4-says-
das/article9154818.ece
*Clarifications*
8. The annual reports ‘Education at a Glance’ Quiz, Q4, September
and the ‘Global Education Monitoring Report’ th
24 (Click Here): Annexure D and E in the
are respectively issued by?
treaty provide for the exceptions to the rule
(a) Organisation for Economic Cooperation that India shall permit unrestricted flow of the
and Development (OECD) and the Global Western rivers of the Indus River system.
Partnership for Education (GPE) Article III to the treaty states, “Except as
(b) OECD and the United Nations provided in Annexure D and E, India shall not
Educational, Scientific and Cultural store any water of, or construct any storage
Organisation (UNESCO) works on, the Western Rivers.” Annexure E is
(c) UNESCO and GPE titled, “Storage of Waters by India on the
Western Rivers”. Click here to see the Treaty
(d) Both are issued by OECD. Text. The answer does not change.

Solution: b. Q3, Quiz, September 27th (Click Here): The


‘Nightingale of India’ usually refers to Sarojini
“A broader issue mentioned in the report Naidu. This is a minor issue. Consider the first
(Education at a glance 2016), which covers
statement as “M.S. Subbulakshmi was born
besides OECD members, partner-countries
including India, is the likelihood that states here”, instead of “The Nightingale of India was
may not be able to realise the 2030 born here”.
Sustainable Development Goal pertaining to
the provision of quality education. This concern
is echoed in the Unesco 2016 report.” Q5, Quiz, September 25th &
“OECD’s annual Education at a Glance looks 26th (Click Here): The penalty is 7.5%, not 45%.
at who participates in education, what is spent Please read the explanation carefully.
on it, how education systems operate and the
results achieved. The latter includes indicators
on a wide range of outcomes, from
Q6, Quiz, September 25th &
comparisons of students’ performance in key
subject areas to the impact of education on 26th (Click Here): Special favours are not
earnings and on adults’ chances of granted under MFN status. Hence option ‘b’ is
employment.” wrong.
“The Global Education Monitoring Report (the
GEM Report, formerly known as the Education
for All Global Monitoring Report) is an
editorially independent, authoritative and
evidence-based annual report published by
UNESCO. Its mandate is to monitor progress
towards the education targets in the new
Sustainable Development Goals (SDGs)
framework.”

©www.InisightsIAS.com 50 ©www.InsightsonIndia.com
|

1. ‘Visaranai’ is India’s official entry for Oscars Select the correct answer using the code given
2017 in the foreign language film category. below:
Which among the following issues does this
movie highlight? (a) 1 only

(a) Migrants being subject to police brutality (b) 2 and 3 only


and corruption. (c) 1, 2 and 3
(b) Social stigma and ostracism faced by (d) None of the above
Leprosy-affected individuals.

(c) Female foeticide.


Solution: b.
(d) Farmers’ suicides.
Poly-Metallic Nodules (Manganese Nodules)
are found in the Central Indian Ocean Basin
Solution: a. region.

‘Visaranai’ translates to ‘Interrogation’. “The ISA earlier approved an application


submitted by the Ministry of Earth Sciences
“The film won the Amnesty International Italia for allotment of 10,000 sq. km. area along with
Award for focusing on human rights violations 15-year plan of work for exploration of PMS
after it premiered in the Orrizonti section at along Central Indian Ridge (CIR) and
the Venice International Festival this year, the Southwest Indian Ridge (SWIR) region of the
first ever for a Tamil language film. Visaranai Indian Ocean. The Union Cabinet approved
is set in the border area of Andhra Pradesh signing of this 15-year contract by the Ministry
where four innocent migrant workers from the with the ISA in its meeting held on June 15,
neighbouring state of Tamil Nadu are callously 2016.”
picked up. The four are tortured into taking
the rap for a crime they have not committed as http://pib.nic.in/newsite/PrintRelease.aspx?
the police find it easier to make scapegoats of relid=151130
the poor than searching for the real
culprits.”….”
3. Which of the following statements with
http://www.bbc.com/news/world-asia-india- reference to Poly-Metallic Sulphides (PMS)
37469678 is/are correct?
http://www.thehindu.com/features/cinema/ 1. PMS contain platinum, gold, and silver.
visaranai-is-indias-official-entry-for-oscars-
2017-in-foreign-language-film- 2. Many such PMS deposits are associated
category/article9136134.ece with hydrothermal vents which host
thriving biological communities.

Select the correct answer using the code given


2. The Government of India through the industry below:
of Earth Sciences recently signed a contract
with the International Seabed Authority for (a) 1 Only
exploration of Poly-Metallic Sulphides (PMS). (b) 2 Only
The contract permits India to explore PMS
along which of the following regions? (c) Both 1 and 2

1. Central Indian Ocean Basin (d) Neither 1 nor 2

2. Central Indian Ridge in the Indian Ocean

3. Southwest Indian Ridge in the Indian Solution: b.


Ocean  “Deep seabed Poly-Metallic Sulphides
(PMS) containing iron, copper, zinc,
silver, gold, platinum in variable

©www.InisightsIAS.com 51 ©www.InsightsonIndia.com
|

constitutions are precipitates of hot fluids Solution: b


from upwelling hot magma from deep
interior of the oceanic crust discharged “Hydrothermal vent fields are hosts of thriving
through mineralized chimneys. PMS in the biological communities such as giant tube
Ocean Ridges have attracted worldwide worms, clam shells, crabs and micro fauna.
attention for their long term commercial as These animals use chemosynthesis – they
well as strategic values.” derive energy from bacterial oxidation of
chemicals in the vent fluids for survival.”
 Point to note: The ISA website does not
mention Platinum as one of its  “The animals inhabiting deep-sea
constituents. But since PIB and some hydrothermal vents are unlike any other
newspapers include the metal, consider it life on Earth. Instead of sunlight, vent life
to be one constituent of PMS. relies on hydrogen sulfide – more
 The next question on ‘Chemosynthesis’ is commonly known as rotten egg gas and
associated with this point: “More than 350 toxic to most land-based life.”
sites of high-temperature hydrothermal  “In a process called chemosynthesis,
vents and about two hundred sites of specialized bacteria create energy from the
significant massive sulphide accumulation hydrogen sulfide present in the mineral-
have been identified since the discovery of rich water pouring out of the vents. These
black smokers in 1978-79. Most of these bacteria form the bottom level of the food
deposits are associated with the mid-ocean chain in these ecosystems, upon which all
ridges, backarc environments, submarine other vent animals are dependent.”
volcanic arcs and in basins near volcanic  “Vent microorganisms are unique in other
arcs. Unusually high temperature and ways, too. They can survive temperatures
thriving biological communities are of up to 113°C.”
most conspicuous of the vents
phenomenon.” Primary Reference:
http://wwf.panda.org/about_our_earth/blue
https://www.isa.org.jm/mineral- _planet/deep_sea/vents_seeps/
resources/56
Improvisation/Source: ISA, PIB;
http://pib.nic.in/newsite/PrintRelease.aspx?
relid=151130

5. Which among the following list is/are


members of the South Asia Sub-regional
4. Consider the following statements with Economic Cooperation (SASEC) program?
reference to ‘Chemosynthesis’:
1. Afghanistan
1. It is a process used by, among others,
deep-sea communities to produce food in 2. Bhutan
absolute darkness.
3. Nepal
2. Some organisms that use the process of
4. Bangladesh
chemosynthesis are known to survive at
temperatures beyond 100 degrees 5. Maldives
Centigrade.
Select the correct answer using the code given
Which of the statements given above is/are below:
correct?
(a) 1, 2, 3 and 4
(a) 1 only
(b) 2, 3, 4 and 5
(b) 2 only
(c) 2, 3 and 4 only
(c) Both 1 and 2
(d) 5 only
(d) Neither 1 nor 2

©www.InisightsIAS.com 52 ©www.InsightsonIndia.com
|

Solution: b. “An Indian strategic thinker of the 3rd century


B.C. bearing the name Kautilya dwelt on the
“The six (6) member countries of the South subject of threats that states encounter.”
Asia Sub-regional Economic Cooperation
(SASEC) program—Bangladesh, Bhutan, He is traditionally identified as Kauṭilya or
India, Maldives, Nepal, and Sri Lanka— Vishnugupta, who authored the ancient
released this week, the SASEC Operational Indian political treatise, the Arthashastra.
Plan (OP) 2016-2025. The SASEC OP is the “Kautilya (Click Here) became a counsellor and
program’s first comprehensive long-term plan adviser to Chandragupta (reigned c. 321–c.
to promote greater economic cooperation 297), founder of the Mauryan empire of
among the member countries.” northern India, but lived by himself. He was
instrumental in helping Chandragupta
http://pib.nic.in/newsite/PrintRelease.aspx? overthrow the powerful Nanda dynasty at
relid=151132 Pataliputra, in the Magadha region.”

Improvisation:
6. Which among the following serves as the http://pib.nic.in/newsite/PrintRelease.aspx?
secretariat for the SASEC program? relid=151207

(a) India

(b) Bangladesh 8. In which of the following ways will the


execution of ‘Project SAKSHAM’ help?
(c) The Asian Development Bank
(a) Implementation of the Goods and Services
(d) None of the above Tax.

(b) Enabling Nigerian officers to become


effective and competent in counter-terror
Solution: c.
operations.
“Established in 2001, the SASEC program is a
(c) Revival of sick Central Public Sector
project-based partnership to promote regional
Enterprises.
prosperity by improving cross-border
connectivity, boosting trade among member (d) Improve India’s ranking in the ‘Global
countries, and strengthening regional Competitiveness Index’ within the next 2
economic cooperation. The Asian Development years so as to find itself in the top ten.
Bank (ADB) serves as the secretariat and lead
financier of the SASEC program.”

http://pib.nic.in/newsite/PrintRelease.aspx? Solution: a.
relid=151132 “The Cabinet Committee on Economic Affairs,
chaired by the Prime Minister Shri Narendra
Modi has approved ‘Project SAKSHAM’, a
7. The Indian teacher, philosopher, economist, New Indirect Tax Network (Systems
jurist and royal advisor ‘Vishnugupta’, is Integration) of the Central Board of Excise and
believed to have assisted who among the Customs (CBEC). Introduction of GST will
following in their rise to power? result in a several-fold increase in the number
of taxpayers and resultant document load on
(a) Kanishka
the system. CBEC’s current IT system was set
(b) Asoka up in 2008. It cannot cater to the increased
load under GST without an immediate
(c) Samudragupta upgrade of its IT Infrastructure.”
(d) Chandragupta Maurya “It will help in:

 implementation of Goods and Services Tax


Solution: d. (GST),

©www.InisightsIAS.com 53 ©www.InsightsonIndia.com
|

 extension of the Indian Customs Single


Window Interface for Facilitating Trade
(SWIFT) and
NOTES
 other taxpayer-friendly initiatives under
Digital India and Ease of Doing Business
of Central Board of Excise and Customs.”

http://pib.nic.in/newsite/PrintRelease.aspx?
relid=151194

9. The expression “Green Gold” refers to

(a) Bamboo

(b) Solar Panels

(c) Gas Hydrates

(d) Tendu leaves

Solution: a.

“Delivering the inaugural address, Dr Jitendra


Singh called for a “green gold” revolution
through promotion and trade of bamboo
which, he said, had a vast unexplored
potential to bring about a quantum jump in
the economic growth of Northeast in
particular and whole of India in general. The
expression “green gold” has been coined to
refer to bamboo because it is a miracle plant
which is often placed in the category of grass
product rather than forest product and
matures faster than its other peers, he added.
He said that there is approximately 8.96
million hectare of forest area of this country
which is covered by bamboo and Northeast is
the home for over 60% of India’s bamboo
reserves.”

http://pib.nic.in/newsite/PrintRelease.aspx?
relid=151167

©www.InisightsIAS.com 54 ©www.InsightsonIndia.com
|

2. The research project, known as ‘NAWDEX’, will


*Clarifications* investigate the

(a) Impact of contrails on cloud formation.


Q3, Q4, September 29th (Click Here): A
mistake we made while uploading the quiz. (b) Contribution of airplanes to climate
The explanation is clear and in both cases the change.
answer must be option ‘c’, that is, both
statements in both questions are correct. (c) Impact of interaction of ash clouds and
airplane emissions on cloud formation.

(d) Influence of the jet stream on patterns of


1. ‘Google Station’, recently seen in the news, is weather down on the surface.
(a) A high-speed wi-fi access across railway
stations.

(b) A research and development project to Solution: d.


provide Internet access to rural and remote “A major international effort is under way to
areas. research one of the greatest unknowns in
(c) An Internet connection speed of up to one weather forecasting – the influence of the jet
gigabit per second for both download and stream. For the first time, a fleet of drones and
upload, being deployed across all central planes is being deployed from the United
government offices. States, Iceland and Britain to investigate the
flow of air crossing the Atlantic. Jet streams
(d) A research and development project to are powerful currents of high-altitude wind
understand the internet usage pattern of that govern the patterns of weather down on
rural residents. the surface”.

“The research project, known as Nawdex,


involves scientists from more than 30
Solution: a. institutes and organisations from more than
“Google is to roll out a comprehensive public half a dozen countries”. The geographical
wi-fi platform in India, as part of its bid to get scope of this project is restricted to North
more people using its services. Dubbed Google America and Europe.
Station, the service will see wi-fi hotspots http://www.bbc.com/news/science-
rolled out in stations, with plans to expand environment-37508156
this to shopping malls and cafes at some point.
The service will be free to start with, but Google
will be looking to monetise it at some point”.
3. With reference to the ‘Tsutsugamushi
“Google already offers high-speed free wi-fi Triangle’, which of the following statements
access at 52 railway stations across India.
Writing on the official Google blog, chief is/are correct?
executive Sundar Pichai said the project
“would rank as the largest public wi-fi project 1. It refers to the region to which the Bush
in India, and among the largest in the world, Typhus disease is endemic.
by number of potential users”.
2. Its vertices are Northern Japan in the
http://www.bbc.com/news/technology-
37483294 North, Northern Australia in the South and
Pakistan in the West.

©www.InisightsIAS.com 55 ©www.InsightsonIndia.com
|

Select the correct answer using the code given 4. ‘Santhara’ refers to
below:
(a) A form of painting recently discovered and
(a) 1 only was practiced by the Early Rajputs.
(b) 2 only (b) The Jain practice of a ritualistic fast unto
(c) Both 1 and 2 death.

(d) Neither 1 nor 2 (c) A Ministry of Women and Child


Development initiative to identify potential
women entrepreneurs, provide them
Solution: c. training and financial assistance.

Both statements are correct. (d) Kautilya’s term for what is known today as
a ‘surgical strike’.
 Scrub typhus, also known as bush typhus
because the mites (Leptotrombidium
deliense, commonly known as trombiculid
Solution: b.
mite) that cause it reside in vegetation
predominantly comprising small shrubs, is “The house of the Dugars in the posh
an infectious disease with symptoms Mandeville Gardens of south Kolkata has been
similar to any viral fever. seeing an endless number of visitors for the
 However, it is not caused by a virus, but by past few days. Amidst chanting of Jain
a parasite called Orientia tsutsugamushi, mantras, people trickle in silently and pay
which is transmitted by the bite of infected their respects to an elderly lady who has
mite larvae in soil containing scrub confined herself in one of the rooms of the
vegetation. The disease is more common house. Sohani Devi Dugar, 83, chose to
during the wet season when the mites lay embrace santhara — a Jain religious
eggs. practice of a ritualistic fast unto death.
 Scrub typhus is endemic to a part of the Thursday was the tenth day of santhara for
world known as the “tsutsugamushi Ms. Sohani Devi. Her decision came about
triangle”, which extends from northern after doctors could assure little hope of her
Japan and far-eastern Russia in the north, recovery from an advanced stage of throat
to northern Australia in the south, and to cancer.”
Pakistan in the west.
http://www.thehindu.com/news/cities/kolka
 Scrub typhus is essentially an
ta/83yearold-on-santhara-draws-thousands-
occupational disease among rural
in-kolkata/article9164873.ece
residents in the Asia-Pacific region.
 Scrub typhus, a dreaded disease in pre-
antibiotic era, is a militarily important
disease that caused thousands of cases in 5. The ‘Marrakesh Treaty’
the Far East during the Second World War. (a) Established the World Trade Organisation.
Primary Source: (b) Facilitates access to published works by
http://indianexpress.com/article/explained/ visually impaired persons and persons
scrub-typhus-himachal-pradesh-the-jungle- with print disabilities.
fever-that-is-stalking-himachal-pradesh-
3055135/ (c) Is the first public health treaty of the world,
which deals with control of tobacco
Secondary Source: FAQ, WHO consumption.

(d) a, b and c.

Solution: b.

©www.InisightsIAS.com 56 ©www.InsightsonIndia.com
|

“The Marrakesh VIP Treaty (formally the


Marrakesh Treaty to Facilitate Access to
Published Works by Visually Impaired Persons
and Persons with Print Disabilities, colloquially
MVT) is a treaty on copyright adopted in
Marrakesh, Morocco, on 28 June 2013. The
treaty allows for copyright exceptions to
facilitate the creation of accessible versions of
books and other copyrighted works for visually
impaired persons. The treaty sets a norm for
countries ratifying the treaty to have a http://www.thehindu.com/business/Industr
domestic copyright exception covering these y/opec-agrees-on-modest-oil-production-
activities, and allowing for the import and curbs/article9163898.ece
export of such materials.”

“The Marrakesh treaty is the second


international trade treaty associated with
Marrakesh, the other being the Marrakesh
Agreement which established the World
Trade Organization in 1994.” *Clarifications*
http://www.thehindu.com/opinion/op-
ed/making-books-accessible-to- Q3, September 30th Quiz,
all/article9163855.ece (Click Here): Regarding the Tsutsugamushi
Triangle – Answer does not change. The
triangle is not a perfect geometrical triangle. It
6. ‘OPEC’, or the Organisation of the Petroleum refers to the geographical space which looks
Exporting Countries, recently agreed on like a triangle when mapped. If the words
modest oil output cuts – the first such deal
‘Northern Japan’ were replaced with ‘Far
since 2008. Which of the following nations
is/are not members of OPEC? Eastern Russia’, then too the statement would
be correct. Additionally, if a similar statement
1. Ecuador appears in UPSC, they won’t be testing your
2. Venezuela rote knowledge or the technicality – they’ll
essentially check whether you have an idea of
3. Iran
it or not. That is, they are likely to give the
4. Indonesia statement in the same form as we have, or in
Select the correct answer using the code given
the following forms: ‘the disease is endemic to
below: India’, or, ‘the disease is endemic to South East
Asia’ and so on – you then would know
(a) 1, 2 and 3
definitely whether the statement is correct or
(b) 1 and 2 only not.
(c) 3 only

(d) None of the above.

Solution: d.

OPEC members include, as of July 2016,


Algeria, Angola, Ecuador, Gabon, Indonesia,
Iran, Iraq, Kuwait, Libya, Nigeria, Qatar,
Saudi Arabia, United Arab Emirates, and
Venezuela.

©www.InisightsIAS.com 57 ©www.InsightsonIndia.com

Potrebbero piacerti anche